184
ANTI-INFECTIVE AGENTS 1- Cephalosporin mechanism of action may include: a) Inhibit protein synthesis b) Inhibit cell wall synthesis c) Inhibit synthesis of essential metabolites d) Alter the function of cell membrane permeability e) Inhibit translation of genetic information 2- The bacteriostatic effect of sulfametoxazole may be increased by: a) Penicillin b) Sulbactam c) Clavulanic acid d) Tarzobactam e) Trimetroprim 3-Correct statements regarding Penicillin V may include which of the following? I-β-lactamase sensitive antibiotic – acid stable II- Has a short half-life because is rapidly excreted by glomerular filtration III- Probenecid increase penicillin’s activity a) I only B) III only c) I and II only d) II and III only e) All are correct 4- The only antibiotic available as chew able tablet includes: a) Penicillin b) Amoxicillin c) Ampicillin d) Erythromycin e) Clarithromycin

Que & Ans_take Away

  • Upload
    bashaby

  • View
    125

  • Download
    0

Embed Size (px)

Citation preview

Page 1: Que & Ans_take Away

ANTI-INFECTIVE AGENTS

1- Cephalosporin mechanism of action may include:

a) Inhibit protein synthesis

b) Inhibit cell wall synthesis

c) Inhibit synthesis of essential metabolites

d) Alter the function of cell membrane permeability

e) Inhibit translation of genetic information

2- The bacteriostatic effect of sulfametoxazole may be increased by:

a) Penicillin

b) Sulbactam

c) Clavulanic acid

d) Tarzobactam

e) Trimetroprim

3-Correct statements regarding Penicillin V may include which of the following?

I-β-lactamase sensitive antibiotic – acid stable

II- Has a short half-life because is rapidly excreted by glomerular filtration

III- Probenecid increase penicillin’s activity

a) I only

B) III only

c) I and II only

d) II and III only

e) All are correct

4- The only antibiotic available as chew able tablet includes:

a) Penicillin

b) Amoxicillin

c) Ampicillin

d) Erythromycin

e) Clarithromycin

Page 2: Que & Ans_take Away

5- Correct statements regarding Erythromycin ESTOLATE may include:

I- Primarily effective against gram-positive bacteria and also have activity against some Gram-negative bacteria

II- It is a macro lid antibiotic very useful as alternative to penicillin for those that have Allergy to penicillin antibiotics

III- Regular erythromycin is better absorbed from GIT that erythromycin estolate

a) I only

b) III only

c) I and II only

d) II and III only

e) All are correct

6- Penicillin should be administrated 1 hour before meals or 2 to 3 hours after meals in order to:

I- Improve the bioavailability

II- Maximize the dissolution rate

III- Avoid interaction with calcium ions

a) I only

b) III only

c) I and II only

d) II and III only

e) All are correct

7-Drugs used in the treatment of meningitis are all of the following, EXCEPT:

a) Penicillins

b) Cephalosporins

c) Gentamycin

d) Streptomycin

e) Sulfonamides

Page 3: Que & Ans_take Away

8- Examples of antibiotic that is an acid stable penicillinase resistant include:

a) Amoxicillin

b) Oxacillin

c) Ampicillin

D) Carbenicillin

e) Penicillin V

9- All of the following penicillins are acid label penicillins, EXCEPT:

a) Penicillin V

b) Penicillin G

c) Methicillin

d) Nafcillin

e) Carbenicillin

10-Which of the following are/is correct regarding Tetracycline?

I- Inhibit protein synthesis

II- May be used in dental works

III- Cannot be taken for infants less than 8 years old

a) I only

b) III only

c) I and II only

d) II and III only

e) All are correct

11- Which of the following describes the mechanism of action of sulfametoxazole?

a) Inhibit protein synthesis

b) Inhibit cell growth by competitive antagonism with folic acid synthesis

c) Inhibit synthesis of essential metabolites

d) Alter the function of cell membrane permeability

e) Inhibit translation of genetic information

Page 4: Que & Ans_take Away

12- Mechanism of inhibit protein synthesis is characteristic of all the following antibiotics, EXCEPT:

a) Aminoglycosides

b) Tetracycline

c) Cephalexin

d) Chloramphenicol

e) Clindamycin

13- Which of the following agents may interfere with the regular effect of penicillins?

a) Antacids

b) Food

c) Probenecid

d) Aminoglycosides

e) All of the above

14- Which of the following drugs cannot be taken orally?

a) INH

b) PASA

c) ASA

d) Streptomycin

e) Oxacillin

15- Example of drug used in prophylaxis of malaria includes:

a) Cloroquine

b) Quinine

c) Primaquin

d) Quinine

e) Quinidine

16- Which of the following is NOT an anti-fungal drug?

a) Amphotericin B

b) Metronidazole

c) Griseofulvin

Page 5: Que & Ans_take Away

d) Nystatin

e) Clotrimazole

17- Examples of drugs that may inhibit DNA gyrase may include:

a) Gentamycin

b) Ceftriaxone

c) Norfloxacin

d) Erythromycin

e) Penicillin

18- Which of the following agents may best treat Meningococcal encephalitis?

a) Sulfametoxazole

b) Erythromycin

c) Cephalexin

d) Tetracycline

e) Penicillin

19- Correct statements regarding aminoglycosides may include:

I- Has a half-life of 12hours

II- The effect continue remain after all drug is eliminated

III- The relation plasma concentration and half-life is not clear

a) I only

b) III only

c) I and II only

d) II and III only

e) All are correct

20- Which of the following statement is/are correct regarding Gray Babies syndrome?

I- Occurs in premature and new born infants when choramphenicol is administrated during the first few days of life

II- Due to inability of the infant to metabolize the drug because of glucoronyl transferase deficiency

III- Glucoronyl transferase is required to detoxify cloramphenicol

a) I only

b) III only

Page 6: Que & Ans_take Away

c) I and II only

d) II and III only

e) All are correct

21- Correct statements regarding Pellagra include:

I- Caused due to deficiency of niacin

II- Caused by failure in converting tryptophan to niacin

III- Characterized by dermatitis, diarrhea and dementia

a) I only

b) III only

c) I and II only

d) II and III only

e) All are correct

22- Gonorrhea, a sexually transmitted disease may be treated by which of the following agents?

I- Cefixime

II- Ceftriaxone

III- Ciprofloxacin

a) I only

b) III only

c) I and II only

d) II and III only

e) All are correct

23- Nosocomial infections may be best treated by which of the following agents?

I- Cephalosporin 3rd generation

II- Macrolides antibiotics

III- Penicillins

a) I only

b) III only

c) I and II only

d) II and III only

e) All are correct

Page 7: Que & Ans_take Away

24- Viral encephalitis, a CNS infection, may be best be treated by which of the following agents?

a) Benzyl penicillin

b) Ceftriaxone

c) Acyclovir

d) Streptomycin

e) Vancomycin

25- Example of drug(s) used to treat pneumonia includes:

I- Ampicillin combined with aminoglycosides

II-Clarithromycin

III- Levofloxacin

a) I only

b) III only

c) I and II only

d) II and III only

e) All are correct

26- Otitis extern, an ear infection may be best is best treated by:

a) Ciprofloxacin eardrop

b) Gentamicin eardrop

c) Prednisone eardrop

d) Levobunolol eardrop

e) Saline solution

27- Otitis media is an ear infection normally caused by S. pneumoniae and may be best treated by:

a) Amoxicillin

b) Penicillin V

c) Ampicillin

d) Amoxicillin clavulanate

e) Oxacillin

Page 8: Que & Ans_take Away

28- First line treatment for UTI-Urinary Tract Infection may include:

a) Sulfametoxazole + Trimetoprim

b) Amoxicillin clavulanate

c) Penicillin procaine

d) Erythromycin

e) All are very effective in treating UTI

29- Which of the following agents would be used in the treatment of infections caused by Helicobacter pylori as combination therapy?

a) Penicillin, amoxicillin, ampicillin and metronidazole

b) Clarithromycin, amoxicillin, omeprazole and metronidazole

c) Clarithromycin, azithromycin and lansoprazole

d) Ceftriaxone and metronidazole only

e) Combinations of antibiotic should not be used due to resistance.

30- Treatment of large intestine infection mainly caused by food intoxication may include:

I- Lots of fluids

II- Ciprofloxacin

III- Metronidazole

a) I only

b) III only

c) I and II only

d) II and III only

e) All are correct

31- Colitis associated with antibiotic use can best be treated by:

a) Cephalosporins

b) Macrolides

c) Aminoglycosides

d) Vancomycin

e) Penicillin

32- Antibiotics that often cause colitis-acute inflammation of colon as side effect include:

I- Broad-spectrum penicillins

Page 9: Que & Ans_take Away

II- Cephalosporins

III- Clindamycin

a) I only

b) III only

c) I and II only

d) II and III only

e) All are correct

33- Drug used together with chloroquine in the treatment of malaria with the purpose to kill persistants parasites in the liver of a person infected with Plasmodium vivax is:

a) Quinine

b) Pyrimethamine

c) Sulfadoxine

d) Primaquine

e) Quinidine

34- Patient is resistant to chloroquine and need to be treated for malaria. Which of the following drugs or combination should be more appropriate for this patient?

a) Atovaquone only

b) Chloroquine and primaquine

c) Quinine and doxacycline

d) Mefloquine

e) C and D are correct

35- Correct statements regarding antibiotics may include:

a) Drugs derivated from synthetic substances that can kill or inhibit bacterial growth

b) Drugs derivated from natural source that can kill or inhibit bacteria growth

c) Drug with ability only to kill bacteria

d) Drug with ability only to inhibit bacterial growth

e) All are wrong regarding antibiotics

36- Drugs derivated from synthetic substances that can kill bacteria is normally called:

Page 10: Que & Ans_take Away

a) Bacteriostatic antibiotic

b) Bactericidal antibiotic

c) Bactericidal antimicrobial

d) Bacteriostatic antimicrobial

e) Bacteriostatic bactericidal antimicrobial

37- Antiviral drugs act in many different ways, they can be classified in:

a) RNA inhibitors only

b) DNA inhibitors only

c) RNA and DNA inhibitors

d) Protease inhibitors

e) All are correct

38- Examples of very narrow spectrum penicillins include all of the following, EXCEPT:

a) Amoxicillin

b) Nafcillin

c) Methicillin

d) Oxacillin

e) All are very narrow spectrum penicillins

39- Examples of extended spectrum penicillin may include:

a) Amoxicillin

b) Ampicillin

c) Cloxacillin

d) Carbenicillin

e) Dicloxacillin

40- Resistance is the major problem with penicillins due to ability of bacteria to breakdown the lactam ring of penicillin and inactivates the drug. To overcome these problem penicillins can be administrated together with β-lactamase inhibitors.

Examples of β-lactamase inhibitors is/are:

I- Clavulanic acid

Page 11: Que & Ans_take Away

II- Sulbactam

III- Tazobactam

a) I only

b) III only

c) I and II only

d) II and III only

e) All are correct

41- Correct statements regarding CEPHALOSPORINS include:

I- Bactericidal antimicrobial

II- Derivated from natural source: Cephalosporium acremonium

III- Broad-spectrum agent

a) I only

b) III only

c) I and II only

d) II and III only

e) All are correct

42- Example of fourth generation cephalosporin includes:

a) Ceftriaxone

b) Cefipime

c) Cefixime

d) Cefotaxime

e) Cefoxitin

43- Correct statement regarding the mechanism of action of Vancomycin may include:

a) Inhibit release of tRNA bound to peptyl site after peptide bond formation

b) Inhibit peptide bound formation between acylaminoacids bound to the peptyl and acceptor site

c) Inhibit stage II of cell wall synthesis preventing polymerization of the linear peptideoglycan

d) Inhibit DNA gyrase in the bacteria

e) Bind to ergosterol from pores resulting in linkage of cellular *******s

Page 12: Que & Ans_take Away

44- Which of the following is one of the most common side effects of vancomycin?

a) Ototoxicity

b) Disulfiram like side effect

c) Diarrhea

d) Weight gain

e) Hallucinations

45- All of the following are narrow spectrum protein synthesis inhibitors, EXCEPT:

a) Erythromycin

b) Azithromycin

c) Clarithromycin

d) Tetracycline

e) Clindamycin

46- Which of the following is NOT a side effect of tetracycline?

a) Alteration in gut flora

b) Disulfiram like reactions

c) Teeth permanent discoloration

d) Fanconi like syndrome

e) Hyperchloremic metabolic acidosis

47- Which of the following agents can be classified as macrolide antibiotics?

I- Azithromycin

II- Clarithromycin

III- Erythromycin

a) I only

b) III only

c) I and II only

d) II and III only

e) All are correct

Page 13: Que & Ans_take Away

48- Wrong statement regarding MACROLIDES ANTIBIOTICS include:

a) Used in both gram-positive and gram-negative infections

b) Used as an alternative to penicillin when patient has allergy to penicillins

c) Inhibit cell wall synthesis of microorganisms

d) Increase gastric motility causing cramps and diarrhea as major side effect

e) Very safe for children

49- All of the following are examples of aminoglycosides drugs, EXCEPT:

a) Nystatin

b) Gentamicin

c) Amikacin

d) Streptomycin

e) Garamycin

50- Which of the following is an anti-infective agent with lowest therapeutic index?

a) Macrolide

b) Cephalosporins

c) Cloramphenicol

d) Aminoglycosides

e) Antifungal agents

51- Fluoroquinolone are anti-infective agents analogs of:

a) PABA

b) Folic acid

c) Vitamin K

d) Cyanide

e) Nalidixic acid

52- Which of the following best classify metronidazole?

a) Antiprotozoa

b) Antibiotic

c) Antifungal

d) Antiviral

Page 14: Que & Ans_take Away

e) Antimicrobial

53- Example of drug(s) used in treatment of tuberculosis include:

I- Isoniazid

II- Rifampin

III- Ethambutol

a) I only

b) III only

c) I and II only

d) II and III only

e) All are correct

54- Drug of choice in the treatment of percutaneous blood infection-schistosomyasis may include:

a) Niclosamide

b) Prazinquatel

c) Mebendazole

d) Dapsone

e) Thiabendazole

55- All are examples of antihelmintic drugs, EXCEPT:

a) Niclosamide

b) Prazinquatel

c) Mebendazole

d) Dapsone

e) Thiabendazole

56- Which of the following is considered a polyene antifungal agent?

a) Miconazole

b) Clotrimazole

c) Amphotericin

d) Itraconazole

e) Ketoconazole

Page 15: Que & Ans_take Away

57- Wrong statement regarding antifungal agents includes which of the following?

a) Polyene antifungal act binding to ergosterol from pores resulting in linkage of cellular *******s

b) Azoles antifungal act blocking the synthesis of ergosterol.

c) Polyene antifungal has the widest spectrum compared to any other agent

d) Side effects of antifungals include fever, chills, anorexia, headaches and muscle pain

e) Side effects of antifungals cannot be treat for Tylenol due to high drug interaction.

58- Which of the following statement is RIGHT regarding influenza treatment?

I- Type A influenza is best treated by amantadine and rimantadine

II- Type B influenza is best treated by oseltamivir and zenamivir

III- Influenza should not be treated with antiviral agents

a) I only

b) III only

c) I and II only

d) II and III only

e) All are correct

59- All of the following are examples of quinolone antimicrobial agents, EXCEPT

a) Cephalexin

b) Ofloxacin

c) Norfloxacin

d) Ciprofloxacin

e) Levofloxacin

60- Which of the following conditions are quinolone antimicrobial drugs mainly used?

a) Respiratory tract infections

b) Urinary tract infections

Page 16: Que & Ans_take Away

c) Gastrointestinal tract infections

d) Endocarditis

e) Cerebral infections

61- Which of the following statements best describes the action of the antitubercular agent ISONIAZID?

a) Inhibit cell wall synthesis of mycobacterium

b) Inhibit the DNA duplication of mycobacterium

c) Inhibit arachydonic acids that feed mycobacterium

d) Inhibit mycolic acids that are unique for mycobacterium

e) Inhibit protein synthesis of mycobacterium

62- Which of the following complications is considered the gravest side effect associated with isoniazid treatment?

a) Cardiac dysfunction

b) Mental dysfunction

c) Hepatitis

d) Renal complications

e) Discoloration of skin

63- The side effects of isoniazid are potentialized by deficiency of which vitamin?

a) Thiamine

b) Pyridoxine

c) Folic acid

d) Niacin

e) Cyanocobalamin

64- All of the following are examples of amino penicillin, EXCEPT:

a) Ampicillin

b) Cloxacillin

c) Amoxicillin

d) Bicampicilin

e) All are correct

65- Which of the following is an antiviral drug used in the treatment of AIDS-HIV?

Page 17: Que & Ans_take Away

I- Acyclovir

II- Zidovudine

III- Stavudine

a) I only

b) III only

c) I and II only

d) II and III only

e) All are correct

66- Bacterial resistance to an antibiotic may occurs due to:

I- Mutation

II- Adaptation

III- Gene transfer

a) I only

b) III only

c) I and II only

d) II and III only

e) All are correct

67- The effectiveness of VANCOMYCIN is seen ONLY in:

a) Virus

b) Gram-negative bacterias

c) Gram-positive bacterias

d) Both gram-positive and negative bacterias

e) Plasmodium

68- Protein synthesis inhibitors may bind to:

a) 30s ribosomal subunit only

b) 50s ribosomal subunit only

c) 70s ribosomal subunit only

d) 30s and 50s and 70s ribosomal subunits

e) 30s and 70s ribosomal subunits

Page 18: Que & Ans_take Away

69- Broad-spectrum antibiotic that may cause severe impairment of bone marrow include which of the following agents?

a) Tetracycline

b) Aminoglycosides

c) Vancomycin

d) Cloramphenicol

e) Clindamycin

70- Which of the following agent is classified as lincosamide antibiotic?

a) Tetracycline

b) Aminoglycosides

c) Vancomycin

d) Cloramphenicol

e) Clindamycin

ANSWERS

1- B

2- E

3- E

4- B

5- C

6- A

7- D

8- B

9- B

10- E

11- B

12- C

13- E

14- D

15- A

Page 19: Que & Ans_take Away

16- B

17- C

18- E

19- D

20- E

21- E

22- E

23- A

24- C

25- E

26- B

27- D

28- A

29- B

30- E

31- D

32- E

33- D

34- E.

35- B

36- C

37- E

38- A

39- D

40- E

41- D

42- B

43- C

44- A

45- D

46- B

47- E

48- C

Page 20: Que & Ans_take Away

49- A

50- D

51- E

52- A

53- E

54- B

55- D

56- C

57- E

58- C

59- C

60- B

61- D

62- C

63- B

64- B

65- D

66- E

67- C

68- D

69- D

70- E

(2)

BIOCHEMISTRY

1-Glycine reaction is normally mediated by:

a) Acetyl Coenzyme A

b) UDP Glucoronyl tranferase

Page 21: Que & Ans_take Away

c) PAPS- Phosphoadenosine-5-Phosphosulfate

d) Sulfotranferase

e) GST-Glutathione S-Transferase

2- Abundant element in the interstitial fluid may include:

a) Potassium

b) Sodium

c) Magnesium

d) Calcium

e) Chloro

3- Which is responsible for N in vivo?

a) Argenine

b) Tryptophan

c) Guanine

d) Thiamine

e) Leucine

4- Enzyme that enables DNA fragments from different sources to be joined:

a) DNA polymerase

b) DNA gyrase

c) DNA ligase

d) RNA transferase

e) None of the above

5- Which of the followings are types of RNA?

I- RNAt - RNA transferase

II- RNAm – RNA mensager

III-RNAr – RNA ribossome

a) I only

b) III only

c) I and II only

Page 22: Que & Ans_take Away

d) II and III only

e) All are correct

6. Which vitamin is/are fat-soluble?

I- Vitamin E

II- Vitamin K

III- Vitamin B

a) I only

b) III only

c) I and II only

d) II and III only

e) All are correct

7-Which of the following vitamin is not water-soluble?

a) Vitamin B

b) Vitamin C

c) Vitamin D

d) Folic acid

e) Riboflavin

8- The most abundant metal ion in human body

a) Iron

b) Magnesium

c) Aluminum

d) Phosphorous

e) Potassium

9- Which of the following is considered the normal potassium serum in human body?

a) 2.0 to 3.5mEq/L

b) 3.5 to 5.0mEq/L

c) 5.0 to 7.5mEq/L

d) 3.5 to 7.5mEq/L

Page 23: Que & Ans_take Away

e) 3.0 to 6.0mEq/L

10- Which of the following is considered normal sodium serum in human body?

a) 35 to 47 mEq/L

b) 147 to 150 mEq/L

c) 135 to 147mEq/L

d) 75 to 135 mEq/L

e) 125 to 145 mEq/L

11- Which of the following is considered as essential vitamin for breast-feed babies?

a) Vitamin A

b) Vitamin B

c) Vitamin C

d) Vitamin D

e) Vitamim E

12- Which of the following hormone acts on cell wall?

a) Insulin

b) Prolactin

c) Cortisol

d) Estrogen

e) Alanine

13- Which of the following vitamins has tetrahydrofolate coenzyme activity based on its pteridine ring?

a) Thiamine

b) Vitamin D

c) Vitamin K

d) Folic acid

e) Pyridoxine

14- What is correct regarding the formation of proteins?

a) Formed by condensation of peptic bond

b) Geometric sequence of amino acids

Page 24: Que & Ans_take Away

c) The formation occurs by complexation of amino acids

d) Proteins are formed from carbohydrates

e) None of the above is right

15- Which is the strongest endogenous analgesic-pain producer in human body?

a) Leukotriene

b) Enkephalin

c) Bradykinin

d) Cytokine

e) Angiotensin

16- Which of the following is considered the FIRST precursor of vitamin A formation?

a) Retinoic acid

b) Carotenoids

c) Cobalt

d) Calciferol

e) Pantothenic acid

17- Biological catalysts responsible for supporting almost all of the chemical reactions that maintain human life process:

a) Nucleic acid

b) Amino acids

c) Carbohydrates

d) Oligopeptides

e) Enzymes

18- Which of the following statements is/are correct regarding enzymes?

I-Enzymes are proteins

II- Enzymes are catalyst because they are never altered during a reaction

III- Michaelis-Menten theory describe the enzymatic reactions

a) I only

b) III only

c) I and II only

Page 25: Que & Ans_take Away

d) II and III only

e) All are correct

19- Zwintter ion reacts with a substrate as:

a) Neutral ion

b) Proton acceptor ion

c) Proton donor ion

d) Anionic

e) Cationic

20- Correct statements regarding mutation may include:

I- Cell division

II- Change in genotype

III- Change in DNA replication altering the gene

a) I only

b) III only

c) I and II only

d) II and III only

e) All are correct

21- Which pyrimidine base is not present in DNA?

a) Thiamine

b) Adenine

c) Uracil

d) Cytosine

e) Guanine

22- The codon sequence Adenine-Guanine-Cytosine-Uracil is seen in

I- RNAs

II- DNAs

III- Both DNAs and RNAs

a) I only

b) III only

Page 26: Que & Ans_take Away

c) I and II only

d) II and III only

e) All are correct

23- Shaped network present in the cell between nucleus and cell wall:

a) Ribossomes

b) Endoplasmic reticulum

c) Mitocondria

d) Cell membrane

e) Nucleotide

24- The Michaelis-Menten equation-Enzymatic reactions will appears ZERO order when:

a) The substrate concentration is much smaller than Km

b) When Km is much smaller than the substrate concentration

c) When Vmax is much smaller than Km

d) When Vmax is much larger than Km

e) When Km approaches Vmax

25- Which vitamin can be classified as hormone?

a) Vitamin D

b) Vitamin D3

c) Vitamin E

d) Vitamin K

e) Vitamin B12

26- False statement regarding vitamin D3 metabolism include:

a) Vitamin C facilitates absorption of vitamin D3 in the liver

b) Hydroxylation of D3 is in kidney

c) Hydroxylation of is D3 in the liver

d) Parathyroid hormone activate metabolism of vitamin D3 in the kidney

e) Skin activates synthesis of vitamin D3 by sunlight

27- Hydrolysis of fixed oils may yields:

Page 27: Que & Ans_take Away

a) Saturated + Unsaturated acids

b) Glycerol + Fatty acids

c) Glucose + Fatty acids

d) Fatty acids + Triglycerides

e) None of the above

28- Example of compound that is phospholipids include:

a) Glycogen

b) Leucine

c) Lecithin

d) Hyaluronic acid

e) Phenylalanine

29- In an enzymatic reaction, addition of competitive inhibitor leads to:

a) Increase the rate of reaction

b) Decrease the rate of reaction

c) Same rate of reaction

d) Decrease rate and increase substrate concentration

e) Decrease both rate and substrate concentration

30- Which of the following is the end product of ANAEROBIC reactions?

a) Pyruvic acid

b) CO2

c) CO2 + H2O

d) Lactic acid

e) Proteins

31- Which of the following is the end product of AEROBIC reactions?

a) Pyruvic acid

b) CO2

Page 28: Que & Ans_take Away

c) CO2 + H2O

d) Lactic acid

e) Proteins

32- Which of the following is an essential amino acid?

a) Histadiol

b) Cysteine

c) Methionine

d) Aspartene

e) Glutamine

33- Which of the following is NOT an essential amino acid?

a) Methionine

b) Argenine

c) Leucine

d) Tryptophan

e) Glycine

34- Which of the following is the end product of proteins?

a) Amino acids

b) Urea

c) Uric acid

d) Purine

e) Phosphorous

35-Purine may be the end product of:

a) Amines

b) Amino acids

c) Uric acid

d) Proteins

e) Enzymes

36- In the typical cell, mitochondria may contain:

Page 29: Que & Ans_take Away

I- Enzymes of Kreb’s cycle

II- Enzymes of respiratory pathway as well cytochrome oxidase

III- Enzymes of fatty acid cycle

a) I only

b) III only

c) I and II only

d) II and III only

e) All are correct

37- Which of the following is the main carbohydrate of body blood?

a) Sorbitol

b) Mannitol

c) Frutase

d) Glucase

e) Lactase

38- The presence of mitochondria in the living cell can be detected by:

a) Neutral reagent

b) Jenus green

c) Morquis reagent

d) Methylene blue

e) Phenolphthalein

39- Which of the following is the only disaccharide NOT synthesized during human metabolism?

a) Lactose

b) Maltase

c) Fructose

d) Glucose

e) Cellulose

40- The conversion of β-carotene to vitamin A is carried out in:

a) Pancreas

b) Spleen

Page 30: Que & Ans_take Away

c) Liver

d) Adrenal cortex

e) Nefrons

41- Keratin is an example of:

a) Protein

b) Peptin

c) Scleroprotein

d) Albuminoid

e) C and D are correct

42- The metabolic degradation of hemoglobin takes place principally in:

a) RBCs

b) Reticulum Endothelial

c) Liver

d) Kidney

e) Pancreas

43- Degradation of hemoglobin in mammal involves:

a) Formation of bile pigments

b) Formation of urobilinogen

c) Formation of biliverdin

d) Non-oxidative cleavage of phosphirin ring

e) All are correct

44- Non-protein portion of hemoglobin consist of:

a) Ferrous complex of protophorphirin IX

b) 3 Heme unit surrounding an iron atom

c) 4 Heme unit surrounding a ferric ion

d) 4 Pyrrole rings linked through a Fe³+ mol.

e) All are correct

45- Which of the following amino acid is an important precursor of hemoglobin?

Page 31: Que & Ans_take Away

a) Alanine

b) Proline

c) Leucine

d) Glycin

e) Histadin

46- Which of the following is an important enzymatic reaction involved in muscular contraction?

a) Glucose-6-phosphatase reaction

b) Glycogenolysis

c) ATP-Creatine phosphokinase reaction

d) Enolase reaction

e) Glucogenolysis

47- Protein disnaturation may happen due to:

a) Exposure to air

b) Shaking

c) Increase in temperature

d) Chemical reagents

e) All are correct

48- Which of the following enzyme is responsible for the synthesis of DNA?

a) DNA polymerase

b) DNA gyrase

c) DNA ligase

d) RNA transferase

e) None of the above

49- Which pyrimidine base is not present in RNA sequence?

a) Uracil

b) Thiamine

c) Cytosine

d) Guanine

e) Adenine

Page 32: Que & Ans_take Away

50- Which of the following is the predominant intracellular element?

a) Sodium

b) Magnesium

c) Potassium

d) Calcium

e) Phosphorous

51- Which of the following is the only polysaccharide NOT synthesized during human metabolism?

a) Maltase

b) Sucrose

c) Glucose

d) Cellulose

e) Frutose

52- Heparin synthesized in the body and can be classified as:

a) Monosaccharideo

b) Polysaccharide

c) Disaccharide

d) Oligosaccharide

e) Amino acid

53- Sucrose is a disaccharide formed from two molecules of monosaccharide:

a) Glucose + frutose

b) Glucose + glucose

c) Glucose + galactose

d) Frutose + frutose

e) Galactose + galactose

54- Enzymes that catalyze the transfer of one group to other group is called:

a) Oxidoreductase enzymes

b) Transferase enzymes

c) Hydrolyses enzymes

Page 33: Que & Ans_take Away

d) Isomerasis enzymes

e) Lyasis enzymes

55- Catabolism of carbohydrates is a process characterized by:

I- Consume of energy

II- Release stored energy from carbohydrates

III-Glycogenolysis is an example of carbohydrates catabolism reaction

a) I only

b) III only

c) I and II only

d) II and III only

e) All are correct

56- The formation of glucose from non-carbohydrates sources such as lactate and Kreb’s cycle metabolites is an anabolic process called:

a) Glycogenolysis

b) Glycolysis

c) Glycogenesis

d) Gluconeogenesis

e) None of the above

57- GLYCOGENESIS is a process characterized by:

a) The breakdown of glycogen into glucose

b) The transformation of glucose to glycogen

c) The breakdown of sugar into pyruvate

d) The breakdown of sugar into lactate

e) The formation of glucose from non-carbohydrates sources

58- Which of the following reactions is controlled by the pancreatic hormone INSULIN?

a) Glycogenolysis

b) Glycolysis

c) Glycogenesis

d) Gluconeogenesis

Page 34: Que & Ans_take Away

e) Anaerobic glycolysis

59- Correct statements regarding RNA messenger may include:

a) Carrier activated aminoacids to the ribosome

b) Specifies aminoacids sequence templant for protein synthesis

c) Enable DNA fragments to be joined

d) Synthesize RNA

e) Carrier the genetic information for all the other cells

60- Element found in the center of vitamin B12-cyanocobalamin:

a) Iron

b) Zinc

c) Cobalt

d) Magnesium

e) Calcium

61- Beriberi is caused by deficiency of which of the following vitamins?

a) Thiamine

b) Riboflavin

c) Niacin

d) Pyridoxine

e) Folic acid

62- Which of the following is the most important vitamin in pregnancy?

a) Cyanocobalamin

b) Folic acid

c) Pantothenic acid

d) Pyridoxine

e) Niacin

63- Which of the following vitamin is derivated from the amino acid tryptophan?

a) Thiamine

b) Riboflavin

Page 35: Que & Ans_take Away

c) Niacin

d) Pyridoxine

e) Folic acid

64- Which of the following vitamin has its chemical structure related to PABA?

a) Cyanocobalamin

b) Folic acid

c) Pantothenic acid

d) Pyridoxine

e) Niacin

65- Vitamin that requires an intrinsic factor to be carried out to ileum in order to suffer absorption from the small intestine:

a) Cyanocobalamin

b) Folic acid

c) Pantothenic acid

d) Pyridoxine

e) Niacin

66- Which of the following vitamin is the precursor of collagen synthesis?

a) Vitanin A

b) Vitamin B

c) Vitamin C

d) Vitamin D

e) Vitamin E

67- Correct statements concerning vitamin E include:

I- It is a lipid soluble vitamin stored in adipose tissues

II- It is absorbed from the small intestine

III- Widely used in pharmaceutical formulations as antioxidant agent

a) I only

b) III only

c) I and II only

Page 36: Que & Ans_take Away

d) II and III only

e) All are correct

68- Which of the following compounds are considered the building blocks of nucleic acids?

a) Nucleotides

b) Nucleosides

c) Monosaccharides

d) Purines

e) Amino acids

69- Proteins are formed from:

a) Purines

b) Carbohydrates

c) Amino acids

d) Monosaccharides

e) Nucleosides

CORRECT ANSWERS

1- A : Glycine reaction is a reaction mediated by acetyl Coenzyme A.

2- B : Interstitial fluid is situated between parts or in the interspaces of a tissue therefore outside the cells. Sodium is the most abundant element outside the cells therefore in the interstitial fluid.

3- A : Arginine is an essential amino acid responsible for N.

4- C : DNA ligase is an enzyme that enables DNA fragments from different sources to be joined.

5- E : All, RNA transferase, RNA mensager and RNA ribosome are types of RNAs.

6- C : All vitamins B and vitamin C are water-soluble.

7- C : Vitamin A, D, E and K are fat-soluble vitamins.

8- A : Iron is the most abundant metal ion in human body.

9- B : Normal potassium serum in human body range between 3.5 to 5.0mEq/L.

10- C : Normal sodium serum in human body range between 135 to 147mEq/L

11- D : The most essential vitamin for breast-feed babies is vitamin D.

12- A : Insulin is a hormone that acts on cell wall

13- D : Folic acid is a vitamin B9 that has tetrahydrofolate coenzyme activity based on its pteridine ring.

14- A : Proteins are formed by a reaction between amino acids called condensation between peptid bonds.

Page 37: Que & Ans_take Away

15- C : Bradykinin is the strongest endogenous analgesic-pain producer in human body.

16- B : Vitamin A is formed from retinoic acid that is formed from beta-carotene.

17- E : Enzymes are biological catalysts responsible for supporting almost all of the chemical reactions that maintain human life process, and accelerate reactions by lowering the energy of the transition state.

18- E : Enzymes are proteins or biological catalysts responsible for supporting almost all of the chemical reactions. They are called catalyst because are never altered during a reaction. Michaelis-Menten theory describes the enzymatic reactions.

19- A: Zwintter ion is a neutral ion with two charges, positive and negative, that reacts neutrally with a substrate.

20- D : Mutation is a permanent transmissible change in the genetic material, usually in a single gene.

21- C : Uracil is only present in RNAs.

22- A : The codon sequence Adenine-Guanine-Cytosine-Uracil is seen in RNAs only. DNAs differ only in the presence of Thiamine instead of uracil.

23- B : Endoplasmic reticulum is a shaped network present in the cell between nucleus and cell wall.

24- E : The Michaelis-Menten equation-Enzymatic reactions will appears ZERO order when Km approaches Vmax and FIRST order when the substrate concentration is smaller than Km.

25-B : Vitamin D3 man be classified as hormone

26- A: Vitamin C does not interfere with absorption of vitamin D3 in the liver.

27- B : Glycerol and fatty acids are the end products of hydrolysis of fixed oils.

28- C : Lecithin is a phospholipid

29- C : Addition of competitive inhibitor in an enzymatic reaction does not alter the rate of reaction.

30- D : Pyruvic acid is the end product of anaerobic reactions

31- C : CO2 and H2O are the end products of aerobic reactions.

32- C : Histadiol, cysteine, aspartame and glutamine are examples of non-essential amino acids

33- E : Glycine is not an essential amino acid.

34- B : Urea is the end product of proteins.

35- C: Purine is the end product of uric acid.

36- E: Enzymes of Kreb’s cycle, enzymes of respiratory pathway as well cytochrome oxidase and enzymes of fatty acid cycle are present in the mitochondria structure of the cell.

37- D : Glucose is the main carbohydrate present in the human blood.

38- B : Jenus green is a laboratory test that detects the presence of mitochondria in the living cell

39- A: Maltase and cellulose are both carbohydrates not synthesized during human metabolism, however maltase is disaccharide and cellulose is a polysaccharide carbohydrate.

40- C: β-carotene is the precursor of vitamin A. The transformation reaction happens in the liver by oxidation.

41- E : Keratin is a scleroprotein that also can be called albuminoid.

42- B : The metabolic degradation of hemoglobin takes place principally in reticulum Endothelial.

43- E : The degradation of hemoglobin in mammal involve formation of bile pigments, formation of urobilinogen, formation of biliverdin and non-oxidative cleavage of phorphirin ring

Page 38: Que & Ans_take Away

44- A : The non-protein portion of hemoglobin consists of ferrous complex of protophorphirin IX.

45- D : Glycin is a non-essential amino acid and important precursor of hemoglobin.

46- C : ATP-Creatine phosphokinase reaction is the most important enzymatic reaction in muscular contraction.

47- E : Protein derivative products are very sensible and may suffer disnaturation by exposure to air, shaking, increase in temperature and addition of chemical reagents in.

48- A: DNA polymerase is the enzyme responsible for the synthesis of DNA.

49- B : Thiamine is the only pyrimidine base not present in RNA sequence; thiamine is only present in DNAs molecule.

50- C : Potassium is the most predominant intracellular element while sodium is the most predominant extracellular element.

51- D: Maltase and cellulose are both carbohydrates not synthesized during human metabolism, however maltase is disaccharide and cellulose is a polysaccharide carbohydrate.

52- B: Heparin is a polysaccharide carbohydrate naturally synthesized in the body.

53- A: Sucrose is a disaccharide formed from two molecules of monosaccharide, glucose and fructose

54- B: Transferase enzymes catalyzes reactions involved in the transfer of one group to other group

55- D : Catabolism of carbohydrates is a process characterized by release of energy stored from carbohydrates. Glycogenolysis and glycolysis are examples of carbohydrates catabolism reaction

56- D : Gluconeogenesis is the formation of glucose from non-carbohydrates sources such as lactate and Kreb’s cycle metabolites is an anabolic process called.

57- B: Glycogenesis is the transformation of glucose to glycogen. It happens in the liver and skeletal muscle from glucose consumed in the diet its synthesis is controlled by the pancreatic hormone insulin

58- C : Glycogenesis is an anabolic reaction that transforms glucose to glycogen in the liver and it is controlled by insulin.

59- B: RNA messenger specifies amino acids sequence templant for protein synthesis.

60- C: B12-cyanocobalamin has a cobalt molecule in the center of its structural molecule.

61- A: Beriberi is caused by deficiency of thiamine.

62- B: Folic acid is one of the most important vitamin to be taken by women that is planning to get pregnant, during pregnancy and in breast-feeding.

63- C : Niacin is derivate from the amino acid tryptophan.

64- B : Folic acid is a vitamin that has its chemical structure similar to PABA structure.

65- A: Cyanocobalamin is not properly absorbed from the gastro intestinal tract therefore it requires an intrinsic factor to be carried out to ileum in order to suffer absorption from the small intestine

66- C: Vitamin C is the precursor of collagen synthesis.

67- E: Vitamin E is a lipid soluble vitamin stored in adipose tissues and well absorbed from the small intestine. It is widely used in pharmaceutical formulations as antioxidant agent

68- A: Nucleotides are considered the building blocks of nucleic acids.

69- C: Proteins are formed from amino acids by condensation of peptide bond

الخطاب بن عمر مواضيع من :

Page 39: Que & Ans_take Away

الكندي للبورد سؤال 5000

اقتباس مع رد اقتباس مع رد

01-08-2008 09:23 AM #3

الخطاب بن عمر

عضــــو

التسجيل تاريخ : Aug 2008

كينيا : اإلقامة

Male : النوع

22 : المشاركات

0 : التقييم معدل

الخطاب بن عمر is an unknown quantity at this point

6 متواجد غير الخطاب بن عمر حاليا

Share

(3)

Biomedical Sciences (25%)

1 Folic acid has tetrahydrofolate coenzyme activity which is based on

the:

a. pyrimidine ring.

b. purine ring.

c. pyrazine ring.

d. pteridine ring.

e. pyridine ring.

2 Enkephalins are peptides that:

a. have narcotic antagonist activity.

b. exert actions resembling those of opiates.

c. are found only in the central nervous system.

d. cause blood vessel wall relaxation.

e. transmit pain impulses.

3 ?-Carotene is the precursor of:

a. retinol.

Page 40: Que & Ans_take Away

b. thiamine.

c. calciferol.

d. riboflavin.

e. retinoic acid.

4 An enzyme is:

a. an oligopeptide.

b. a nucleic acid.

c. unable to hydrolyse polysaccharides.

d. a catalyst.

e. changed chemically in a reaction.

5 Diphtheria is caused by:

a. Plasmodium.

b. Vibrio.

c. Shigella.

d. Neisseria.

e. Corynebacterium.

6 Identify the correct pathway of blood flow from the small intestine

to the liver:

a. haemorrhoidal vein ? portal vein.

b. mesenteric vein ? portal vein.

c. haemorrhoidal vein? inferior vena cava.

d. mesenteric vein ? hepatic artery.

e. gastric vein ? hepatic vein.

7 The anatomical divisions of the small intestine include the:

I jejunum.

II ileum.

III duodenum.

a. I only

b. III only

c. I and II only

d. II and III only

e. I, II and III

8 Parkinson's disease is characterized by a deficiency of:

Page 41: Que & Ans_take Away

a. acetylcholine in the motor cortex.

b. noradrenaline in the spinal motor neurons.

c. gamma amino butyric acid (GABA) in inhibitory pathways.

d. dopamine in the nigrostriatal pathway.

e. serotonin in the brain stem.

9 In which of the following organs or tissues is the action of

acetylcholine NOT described as "nicotinic"?

I Motor endplates of skeletal muscle

II Celiac ganglia

III Circular muscles of the iris

a. I only

b. III only

c. I and II only

d. II and III only

e. I, II and III

10 The blood volume of the average adult is approximately:

a. 5 L.

b. 10 L.

c. 15 L.

d. 20 L.

e. 25 L.

11 Diseases which are viral infections include:

I poliomyelitis.

II rabies.

III Legionnaires' disease.

a. I only

b. III only

c. I and II only

d. II and III only

e. I, II and III

12 Common etiologic agents of community-acquired pneumonia include:

I Escherichia coli.

II Haemophilus influenzae.

Page 42: Que & Ans_take Away

III Mycoplasma pneumoniae.

a. I only

b. III only

c. I and II only

d. II and III only

e. I, II and III

13 Gamma aminobutyric acid (GABA) is:

I the major inhibitory neurotransmitter in the brain.

II found primarily in interneurons.

III synthesized from glycine.

a. I only

b. III only

c. I and II only

d. II and III only

e. I, II and III

14 All of the following statements are true regarding chronic

inflammation, EXCEPT:

a. it may develop as a granulomatous inflammation.

b. it may be caused by persistent infections such as tuberculosis.

c. it may occur with autoimmune diseases such as rheumatoid

arthritis.

d. large numbers of macrophages are found in the infiltrate.

e. lymphocytes and eosinophils are rarely involved in its

development.

Pharmaceutical Sciences (55%)

15 If the pKa of phenobarbital is 7.4, what approximate fraction of

the drug would be ionized at pH 8.4?

a. 10%

b. 30%

c. 50%

d. 90%

e. 100%

16 All of the following statements concerning the lyophilization of a

Page 43: Que & Ans_take Away

parenteral product are correct, EXCEPT:

a. there is minimal loss of activity in heat labile materials.

b. the liquid must be frozen to below the eutectic temperature.

c. the solute usually forms an amorphous glass.

d. the eutectic temperature is the freezing point of the drug

solution.

e. water is removed from the frozen mixture by sublimation.

17 In cardiovascular physiology, LaPlace's law relates the tension

achieved in a blood vessel wall to:

I the length of the vessel.

II the radius of vessel.

III the intraluminal pressure.

a. I only

b. III only

c. I and II only

d. II and III only

e. I, II and III

18 Concentration of a drug in breast milk exceeds that in plasma, if

the drug:

a. is basic.

b. is protein bound.

c. is acidic but not protein bound.

d. has a small volume of distribution.

e. has a large volume of distribution.

19 Mercapturic acid derivatives in phase II metabolism can result from

reactions of:

a. glutathione conjugates.

b. glucuronide conjugates.

c. glycine conjugates.

d. glutamate conjugates.

e. sulfate conjugates.

20 Tetrodotoxin is a selective inhibitor of the:

a. calcium ion channel.

Page 44: Que & Ans_take Away

b. sodium ion channel.

c. potassium ion channel.

d. chloride ion channel.

e. nicotinic ion channel.

21 Atropine is the primary example of an important class of muscarinic

antagonists which are esters of tropic acid and which contain:

a. aniline and piperidine rings.

b. aniline and piperazine rings.

c. aniline and pyrrolidine rings.

d. pyrrolidine and piperazine rings.

e. pyrrolidine and piperidine rings.

22 Carbon monoxide binds to:

I hemoglobin.

II myoglobin.

III cytochrome oxidase.

a. I only

b. III only

c. I and II only

d. II and III only

e. I, II and III

23 Antioxidants, which mechanistically act as reducing agents,

include:

I ascorbic acid.

II sodium bisulfite.

III citric acid.

a. I only

b. III only

c. I and II only

d. II and III only

e. I, II and III

24 Diazepam Injection U.S.P

Diazepam 5 mg/ml

Ethanol 10%

Page 45: Que & Ans_take Away

Propylene glycol 40%

Benzyl alcohol 1.5%

Water for Injection qs 100%

In the formulation given above, the following ingredient(s) function(s)

as a cosolvent(s):

I ethanol.

II propylene glycol.

III benzyl alcohol.

a. I only

b. III only

c. I and II only

d. II and III only

e. I, II and III

25 Both isoxazole and oxazole rings are found as substituents in some

bacteriostatic sulfanilamides. They are present in these structures

specifically because of their:

a. sizes and shapes.

b. tautomeric activities.

c. enzymatic activities.

d. electron-donating effects.

e. electron-withdrawing effects.

26 The Michaelis-Menten equation will appear first order:

a. when the substrate concentration is much smaller than Km.

b. when Km is much smaller than the substrate concentration.

c. when Vmax is much smaller than Km.

d. when Vmax is much larger than Km.

e. when Km approaches Vmax.

27 With respect to bio*****alence, the parameter “Cmax” is:

a. affected by the extent of absorption only.

b. affected by the rate of absorption only.

c. affected by neither rate nor extent of absorption.

d. affected by both rate and extent of absorption.

e. the only significant parameter.

Page 46: Que & Ans_take Away

28 Which of the hydroxyls in the compound above is a tertiary alcohol?

a. Hydroxyl 1

b. Hydroxyl 2

c. Hydroxyl 3

d. Hydroxyl 4

e. Hydroxyls 1 and 2

29 The above structures are related to one another as:

a. bioisosteres.

b. enantiomers.

c. homologs.

d. rotamers.

e. positional (structural) isomers.

30 Five subjects given a single intravenous dose of a drug have the

following elimination half-lives: 3, 9, 6, 5 and 4 h (hours). The mean

half-life is:

a. 4.0 h.

b. 5.0 h.

c. 5.4 h.

d. 5.8 h.

e. 6.0 h.

31 Reasons for using coatings on tablets include all of the following,

EXCEPT:

a. to mask the taste of the drug.

b. to mask the odor of the drug.

c. to improve the appearance of the tablet.

d. to increase the drug's release rate.

e. to protect the drug from stomach acid.

32 Which of the following factors would be important in determining

the concentration of drug that would be reached in the cerebrospinal

fluid for treatment of meningitis?

I Oil/water partition coefficient

II Binding to plasma protein

III pKa of the drug

Page 47: Que & Ans_take Away

a. I only

b. III only

c. I and II only

d. II and III only

e. I, II and III

33 Which of the following terms indicates a loss of moisture?

a. Deliquescence

b. Efflorescence

c. Hygroscopicity

d. Polymorphism

e. Condensation

34 Physicochemical factors affecting the rate and extent of

percutaneous drug absorption include:

I the partition coefficient of the drug molecule.

II the presence of surfactants in the formulation.

III the type of ointment base.

a. I only

b. III only

c. I and II only

d. II and III only

e. I, II and III

35 Which of the following statements is FALSE regarding tablet

formulation?

a. Diluents are fillers to add bulk to the tablet.

b. Lubricants help the patient to swallow the tablet more easily.

c. Binding agents may be added dry or in solution.

d. Disintegrants draw water into the tablet causing it to burst.

e. Glidants promote the flow of materials during compression.

36 Structure-activity studies of cholinergic drugs indicate that:

I maximum activity is achieved when the quaternary nitrogen is three

carbons removed from the ester oxo group.

II structures which display a quasi-ring conformation exhibit

muscarinic activity.

Page 48: Que & Ans_take Away

III one methyl group of the trimethylammonium group can be replaced

by an ethyl group with only slight loss of activity.

a. I only

b. III only

c. I and II only

d. II and III only

e. I, II and III

37 Which of the following is an azo dye?

a. Hexachlorophene

b. Nitrofurantoin

c. Methenamine

d. Phenazopyridine

e. Nalidixic acid

38 If the total body clearance of a drug is 200 mL/min in a normal

healthy adult and the renal clearance is 10 mL/min then one may assume

that:

a. the drug is extensively metabolized.

b. greater than normal drug accumulation would occur in patients with

moderate renal failure.

c. entero-hepatic recycling is significant.

d. the drug is not bound to plasma proteins.

e. the drug is concentrated in adipose tissue.

39 Approximately 50% of dicloxacillin is excreted unchanged in the

urine. If the normal dosage schedule for dicloxacillin is 125 mg q6h, a

patient with renal function 20% of normal should receive:

a. 25 mg q6h.

b. 31.25 mg q6h.

c. 62.5 mg q6h.

d. 75 mg q6h.

e. 125 mg q12h.

40 Select the drug whose major metabolites have therapeutically

significant pharmacological activity:

a. oxazepam.

Page 49: Que & Ans_take Away

b. nitrazepam.

c. diazepam.

d. triazolam.

e. clonazepam.

41 The major pathway for the biotransformation of the following

compound is by hydrolysis. Identify the site which would be most susceptible

to hydrolysis.

a. A

b. B

c. C

d. D

e. E

42 Biceps muscles are part of:

I the arm.

II the thigh.

III the rib cage.

a. I only

b. III only

c. I and II only

d. II and III only

e. I, II and III

43 The mechanism of antipsychotic effect of phenothiazines is thought

to be due to:

a. improvement of cholinergic transmission.

b. blockade of catecholamine re-uptake.

c. blockade of dopamine receptors.

d. blockade of enkephalinergic neurons.

e. prolactin release.

44 Mechanisms of drug interactions with acetylsalicylic acid (ASA)

include:

a. induction of microsomal enzymes.

b. inhibition of microsomal enzymes.

c. inhibition of liposomal enzymes.

Page 50: Que & Ans_take Away

d. displacement of other drugs bound to serum albumin.

e. enhanced platelet aggregation.

45 All of the following drugs will increase the heart rate directly or

by reflex when injected intravenously into a young healthy adult,

EXCEPT:

a. isoproterenol.

b. histamine.

c. phentolamine.

d. phenylephrine.

e. atropine.

46 Which of the following diuretics is used to block the Na±H+

exchange system of the renal tubule?

a. Furosemide

b. Hydrochlorothiazide

c. Spironolactone

d. Acetazolamide

e. Amiloride

47 Atropine poisoning can be recognized by all of the following signs

or symptoms, EXCEPT:

a. dry skin.

b. flushed appearance.

c. delirium and restlessness.

d. mydriasis.

e. diarrhea.

48 The organophosphates commonly found in insecticides are thought to

act by which of the following mechanisms?

a. Combining with acetylcholine

b. Potentiating the action of acetylcholinesterase

c. Forming a very stable complex with acetylcholinesterase

d. Reacting at the cholinergic receptor

e. Preventing the release of acetylcholine from the nerve ending

49 The skeletal structure of the hipbone includes the:

I ilium.

Page 51: Que & Ans_take Away

II ischium.

III pubis.

a. I only

b. III only

c. I and II only

d. II and III only

e. I, II and III

50 Pernicious anemia is:

a. due to dietary deficiency of vitamin B12.

b. prevented by oral administration of a vitamin B12 supplement.

c. treated by parenteral administration of folic acid.

d. treated by parenteral administration of vitamin B12.

e. caused by dietary iron deficiency.

51 The optic disk is also called the:

a. blind spot.

b. cornea.

c. iris.

d. pupil.

e. macula lutea.

52 Acarbose smooths and lowers:

a. postprandial blood glucose levels by blocking absorption of

carbohydrates.

b. postprandial blood glucose levels by delaying absorption of

carbohydrates.

c. preprandial blood glucose levels by blocking absorption of

carbohydrates.

d. preprandial blood glucose levels by delaying absorption of

carbohydrates.

e. preprandial and postprandial blood glucose levels by blocking

absorption of carbohydrates.

53 Autoimmune disorders include which of the following?

I Graves' disease

II Systemic Lupus Erythematosus

Page 52: Que & Ans_take Away

III Osteoarthritis

a. I only

b. III only

c. I and II only

d. II and III only

e. I, II and III

54 Acute hemorrhagic gastritis is commonly seen with:

a. chronic alcoholism.

b. H. pylori bacterial infections.

c. carcinoma of the stomach.

d. carcinoma of the colon.

e. Travellers' diarrhea.

55 True statements regarding the use of monoclonal antibodies include

which of the following?

I Flu-like symptoms commonly occur at the start of therapy.

II T cells are blocked from initiating the rejection process.

III Orthoclone is a human immunoglobulin product.

a. I only

b. III only

c. I and II only

d. II and III only

e. I, II and III

Questions 56 to 57 inclusive refer to the following:

KJ, a 23 year-old female, is to receive lithium therapy with a regimen

of lithium carbonate 600 mg q8h. The total body clearance of lithium is

0.44 mL/s (1.6 L/h) and the biological half-life is 18 h. The

molecular weight of lithium carbonate (Li2CO3) is 74.

56 The number of millimoles of lithium (Li+1) that KJ will receive

with each dose is:

a. 8

b. 16

c. 24

d. 32

Page 53: Que & Ans_take Away

e. 48

57 The number of milli-*****alents of lithium that KJ will receive

with each dose is:

a. 8

b. 16

c. 24

d. 32

e. 48

58 The fraction of bound receptors in a well-defined drug-receptor

binding system is determined by:

I the equilibrium drug-receptor dissociation constant.

II the free drug concentration.

III the total receptor concentration.

a. I only

b. III only

c. I and II only

d. II and III only

e. I, II and III

59 In some parenteral formulations, sodium metabisulphite is included

as:

a. an antioxidant.

b. a chelator.

c. a solubilizer.

d. a cosolvent.

e. an emulsifier.

60 Potassium-sparing diuretics:

I exert their effect in the proximal tubule.

II may cause intracellular alkalosis.

III include the aldosterone antagonists.

a. I only

b. III only

c. I and II only

d. II and III only

Page 54: Que & Ans_take Away

e. I, II and III

61 Dorzolamide:

I has a sulfonylurea structure.

II has carbonic anhydrase inhibitor activity.

III is useful in the treatment of glaucoma.

a. I only

b. III only

c. I and II only

d. II and III only

e. I, II and III

62 True statements regarding transcription include which of the

following?

I The enzyme responsible for initiating transcription is RNA

polymerase.

II The genetic information contained in the nucleotide sequence of

mRNA is translated into a protein structure.

III The information carried in the gene sequence is transferred to

RNA.

a. I only

b. III only

c. I and II only

d. II and III only

e. I, II and III

63 Which of the following agents may cause pulmonary toxicity?

I Amiodarone

II Bleomycin

III Ramipril

a. I only

b. III only

c. I and II only

d. II and III only

e. I, II and III

64 An angiotensin-converting enzyme (ACE) inhibitor for which both

Page 55: Que & Ans_take Away

renal elimination and metabolism are important in the elimination of the

drug and its active metabolites is:

a. enalapril.

b. fosinopril.

c. lisinopril.

d. quinapril.

e. ramipril.

65 Which of the following agents is NOT associated with the toxic

effect of Stevens-Johnson syndrome?

a. Warfarin

b. Cotrimoxazole

c. Amoxicillin

d. Phenobarbital

e. Lamotrigine

66 Once daily dosing of aminoglycosides is effective due to:

a. prolonged residence of the antibiotic in the body.

b. post-antimicrobial effect.

c. enhanced tissue accumulation.

d. reduced renal clearance.

e. higher peak-trough differences.

67 The method of ionization of mass spectrometry which results in

well-established diagnostic fragmentation patterns that are useful in the

identification of compounds of unknown structure is called:

a. atmospheric pressure chemical ionization (APCI).

b. chemical ionization (CI).

c. electron impact (EI) ionization.

d. electrospray ionization (ESI).

e. fast atom bombardment (FAB) ionization.

Pharmacy Practice (20%)

68 The ethical principle of veracity requires that:

a. we respect the rights of others to make choices.

b. we act with honesty, without deception.

c. we avoid, remove or prevent harm.

Page 56: Que & Ans_take Away

d. we do good to patients.

e. we act with fairness.

69 MT, a pharmacy technician, has worked in a community pharmacy for 5

years. He is well-trained and welcomes new learning opportunities. His

manager, now wanting to delegate a new technical project to him,

should:

I discuss suggested steps to accomplish the project.

II negotiate a time schedule for completion of the project.

III provide support and followup as required.

a. I only

b. III only

c. I and II only

d. II and III only

e. I, II and III

70 Which of the following antacids has cathartic side-effects?

a. Calcium carbonate

b. Dihydroxyaluminum sodium carbonate

c. Magnesium hydroxide

d. Calcium sulfate

e. Aluminum silicate

71 Side effects that may be expected to occur from the administration

of oral contraceptives include all of the following, EXCEPT:

a. fluid retention.

b. headache.

c. hypertension.

d. constipation.

e. depression.

72 In deciding what drugs are appropriate for its formulary, the

hospital must consider a drug's efficacy, associated workload, and

acquisition cost. Several new antifungal IV drugs (drug A, drug B, drug C, and

drug D), all with equal efficacy, have recently become available.

Currently the hospital stocks drug E, which has been available for several

years. Data for the medications is as follows:

Page 57: Que & Ans_take Away

DrugCost/DayDosing FrequencyTreatment Duration

(days)A$2.50QID14B$2.25once daily14C$5.00BID7D$2.25BID7E$2 .25QID7 The most appropriate choice

for the hospital is:

a. Drug A.

b. Drug B.

c. Drug C.

d. Drug D.

e. Drug E.

73 When evaluating patients with angina for possible development of

nitrate tolerance, true statements include:

I tolerance is rarely associated with isosorbide dinitrate.

II tolerance does not occur in patients who are concurrently on

metoprolol.

III tolerance is dependent upon the administration schedule of

nitrate medication.

a. I only

b. III only

c. I and II only

d. II and III only

e. I, II and III

74 Which financial statement could be used to determine the total

value of prescription drug sales for a pharmacy during the course of a

year?

a. Balance sheet

b. Statement of investments

c. Statement of changes in financial position

d. Income statement

e. Statement of equity

75 The Latin abbreviation for "before meals" is:

a. ac.

b. aa.

c. pc.

d. ic.

Page 58: Que & Ans_take Away

e. cc.

76 Erythromycin:

I is effective against gram-positive cocci.

II is a macrolide antibiotic.

III can increase gastrointestinal tract motility.

a. I only

b. III only

c. I and II only

d. II and III only

e. I, II and III

77 Aerosol OT (AOT) is used in veterinary medicine as a laxative. If

250 g of AOT is dissolved in 1 000 mL of glycerin (density of glycerin

is 1.25 g/mL), the concentration of AOT in the solution is:

a. 12.5% w/w.

b. 16.7% w/w.

c. 20.0% w/w.

d. 23.8% w/w.

e. 25.0% w/w.

78 Water used for reconstitution of an additive for a large volume

parenteral should be:

a. sterile water.

b. distilled water.

c. Bacteriostatic Water for Injection (United States Pharmacopeial

standard).

d. Water for Injection (United States Pharmacopeial standard).

e. Sterile Water for Injection (United States Pharmacopeial

standard).

79 The style of management in which the owner of a pharmacy emphasizes

the development of detailed policies and written procedures for

employees to observe at all times is referred to as:

a. autocratic.

b. bureaucratic.

c. participative.

Page 59: Que & Ans_take Away

d. benevolent.

e. consultative.

80 The shock and airway edema of anaphylaxis are best treated with:

a. salbutamol.

b. diphenhydramine.

c. adrenaline.

d. acetazolamide.

e. aminophylline.

81 Sulfinpyrazone is used as a(n):

a. analgesic agent.

b. antipyretic agent.

c. anti-inflammatory agent.

d. agent which increases renal tubular reabsorption of uric acid.

e. uricosuric agent.

82 In the treatment of rheumatoid arthritis, the salicylates:

a. stop and often reverse the progressive joint involvement.

b. specifically reverse the cause of the disease.

c. provide only analgesic and anti-inflammatory effects.

d. are effective because they are uricosuric agents.

e. are more effective when given with allopurinol.

83 Prednisone may produce all of the following effects, EXCEPT:

a. skeletal muscle weakness.

b. hypoglycemia.

c. sodium retention.

d. peptic ulceration.

e. lowered resistance to infection.

84 In an adequately powered, randomized controlled trial conducted

over 2 years, the desired clinical outcome (i.e. prevention of a serious

cardiovascular event) with a new drug is achieved in 25% of the study

sample. In the patients who receive a placebo, only 15% obtain the same

clinical benefit. The relative risk reduction achieved with the new drug

over the study period is:

a. 10%.

Page 60: Que & Ans_take Away

b. 15%.

c. 25%.

d. 40%.

e. 50%.

85 In an adequately powered, randomized controlled trial conducted

over 3 years, a specific serious side effect (i.e. reduction in

leukocytes) with conventional therapy is seen in 0.5% of the study sample. In

patients who receive a newly discovered drug, only 0.45% experience the

same side effect. Based on these results, the minimum number of patients

that would have to receive the new drug for 3 years to statistically

demonstrate the prevention of one episode of this side effect in at least

one patient is:

a. 15.

b. 20.

c. 150.

d. 200.

e. 2000.

86 The pharmacist fills a prescription for sumatriptan 100 mg tablets

for a migraine patient. Appropriate information to provide to the

patient includes which of the following?

a. If the sumatriptan does not relieve the headache within four

hours, ergotamine may be used.

b. If no relief is achieved in two hours, sumatriptan may be

repeated.

c. If the headache is relieved but another headache occurs eight

hours later, sumatriptan may be used for the second headache.

d. The maximum dosage in any 24 hour period is six tablets.

e. If relief is not achieved, no other medication can be used for 24

hours.

87 Auxiliary labels which should be used on cotrimoxazole suspension

include which of the following?

I Shake well before using.

II Take with plenty of fluids.

Page 61: Que & Ans_take Away

III Exposure to sunlight may cause adverse reactions.

a. I only

b. III only

c. I and II only

d. II and III only

e. I, II and III

88 A patient purchasing sublingual nitroglycerin tablets should be

told to store the medication:

a. in the refrigerator.

b. in a plastic vial with a childproof cap.

c. with the original cotton.

d. in an amber glass bottle with a metal cap.

e. in a warm, dry place.

89 Factors that determine bio*****alence of two brands of a drug

include:

I the taste of the preparations.

II the physical appearance of the preparations.

III pharmacokinetic parameters of the preparations.

a. I only

b. III only

c. I and II only

d. II and III only

e. I, II and III

90

Rx Timolol 0.25% drops

Mitte: 15 mL

Sig: gtt. i o.d. BID

On the prescription label the instructions to the patient should read:

a. apply one drop into both eyes twice daily.

b. instil one drop into the left eye twice daily.

c. instil one drop into the right eye twice daily.

d. shake well and instil one drop into the left eye twice daily.

e. shake well and instil one drop into the right ear twice daily.

Page 62: Que & Ans_take Away

91 Following the ethical principle of nonmaleficence requires that

pharmacists:

a. respect the rights of patients to make choices.

b. do good to patients, placing the benefit of the patient above all

else.

c. avoid, remove or prevent harm from people.

d. act with fairness, allowing people access to pharmacy care.

e. act with honesty, without deception.

92 Dopamine is useful in the treatment of cardiogenic shock because

it:

I selectively dilates renal and mesenteric vascular beds.

II does not induce peripheral vasoconstriction.

III decreases the force of myocardial contraction.

a. I only

b. III only

c. I and II only

d. II and III only

e. I, II and III

93 The following prescription is received in a community pharmacy:

Losec® 20 mg

S: Take 1 tab BID

M: 2 weeks

Which of the following is correct labelling for this prescription?

a. Take one tablet twice weekly (28 tablets).

b. Take one tablet twice weekly (4 tablets).

c. Take two tablets twice daily (56 tablets).

d. Take one tablet twice daily (28 tablets).

e. Take one tablet twice daily (14 tablets)

The next section includes an EXAMPLE OF THE CASE SCENARIO FORMAT, in

which there are two or more questions in sequence, that are related to

the patient information provided in the (bolded) stem shown at the top of

the case.

When choosing your response to questions in the case scenario format,

Page 63: Que & Ans_take Away

you must base your choice on the patient-specific information presented

in the case.

Questions 94 to 96 inclusive refer to the following:

SM is a 34 year old female who, while vacationing in Mexico, began

prophylactic treatment for Travellers' Diarrhea. Shortly thereafter she

complained of a feeling of fullness in her ears, black stools and a black

tongue. SM’s previous history includes an allergy to sulfonamides.

94 Which of the following drugs could be the cause of SM's complaints?

a. Bismuth subsalicylate

b. Cotrimoxazole

c. Doxycycline

d. Amoxicillin

e. Loperamide

95 Organisms commonly implicated in the cause of Traveller's Diarrhea

include:

I Bacteroides fragilis.

II Escherichia coli.

III Shigella sp.

a. I only

b. III only

c. I and II only

d. II and III only

e. I, II and III

96 Drugs shown to be effective in the prevention of Travellers'

Diarrhea, which might have been prescribed for SM include:

I doxycycline.

II cotrimoxazole.

III amoxicillin.

a. I only

b. III only

c. I and II only

d. II and III only

e. I, II and III

Page 64: Que & Ans_take Away

Answers

Evaluating Examination Sample Questions

The following sample questions are not intended or designed to be a

sample examination, and do not represent an exact model of the Evaluating

Examination in terms of difficulty and proportion of topics. However,

individually, these questions are intended to be representative, in

format and phrasing style, of the types of questions found on the

Evaluating Examination. They also illustrate a variety of the subject areas

contained in the examination blueprint. Please note that these questions

are reviewed and updated periodically.

Biomedical Sciences

1 Folic acid has tetrahydrofolate coenzyme activity which is based on

the:

a. pyrimidine ring.

b. purine ring.

c. pyrazine ring.

*d. pteridine ring.

e. pyridine ring.

2 Enkephalins are peptides that:

a. have narcotic antagonist activity.

*b. exert actions resembling those of opiates.

c. are found only in the central nervous system.

d. cause blood vessel wall relaxation.

e. transmit pain impulses.

3 ?-Carotene is the precursor of:

*a. retinol.

b. thiamine.

c. calciferol.

d. riboflavin.

e. retinoic acid.

4 An enzyme is:

a. an oligopeptide.

b. a nucleic acid.

Page 65: Que & Ans_take Away

c. unable to hydrolyse polysaccharides.

*d. a catalyst.

e. changed chemically in a reaction.

5 Diphtheria is caused by:

a. Plasmodium.

b. Vibrio.

c. Shigella.

d. Neisseria.

*e. Corynebacterium.

6 Identify the correct pathway of blood flow from the small intestine

to the liver:

a. haemorrhoidal vein ? portal vein.

*b. mesenteric vein ? portal vein.

c. haemorrhoidal vein? inferior vena cava.

d. mesenteric vein ? hepatic artery.

e. gastric vein ? hepatic vein.

7 The anatomical divisions of the small intestine include the:

*I jejunum.

*II ileum.

*III duodenum.

a. I only

b. III only

c. I and II only

d. II and III only

*e. I, II and III

8 Parkinson's disease is characterized by a deficiency of:

a. acetylcholine in the motor cortex.

b. noradrenaline in the spinal motor neurons.

c. gamma amino butyric acid (GABA) in inhibitory pathways.

*d. dopamine in the nigrostriatal pathway.

e. serotonin in the brain stem.

9 In which of the following organs or tissues is the action of

acetylcholine NOT described as "nicotinic"?

Page 66: Que & Ans_take Away

I Motor endplates of skeletal muscle

II Celiac ganglia

*III Circular muscles of the iris

a. I only

*b. III only

c. I and II only

d. II and III only

e. I, II and III

10 The blood volume of the average adult is approximately:

*a. 5 L.

b. 10 L.

c. 15 L.

d. 20 L.

e. 25 L.

11 Diseases which are viral infections include:

*I poliomyelitis.

*II rabies.

III Legionnaires' disease.

a. I only

b. III only

*c. I and II only

d. II and III only

e. I, II and III

12 Common etiologic agents of community-acquired pneumonia include:

I Escherichia coli.

*II Haemophilus influenzae.

*III Mycoplasma pneumoniae.

a. I only

b. III only

c. I and II only

*d. II and III only

e. I, II and III

13 Gamma aminobutyric acid (GABA) is:

Page 67: Que & Ans_take Away

*I the major inhibitory neurotransmitter in the brain.

*II found primarily in interneurons.

III synthesized from glycine.

a. I only

b. III only

*c. I and II only

d. II and III only

e. I, II and III

14 All of the following statements are true regarding chronic

inflammation, EXCEPT:

a. it may develop as a granulomatous inflammation.

b. it may be caused by persistent infections such as tuberculosis.

c. it may occur with autoimmune diseases such as rheumatoid

arthritis.

d. large numbers of macrophages are found in the infiltrate.

*e. lymphocytes and eosinophils are rarely involved in its development.

Pharmaceutical Sciences

15 If the pKa of phenobarbital is 7.4, what approximate fraction of

the drug would be ionized at pH 8.4?

a. 10%

b. 30%

c. 50%

*d. 90%

e. 100%

16 All of the following statements concerning the lyophilization of a

parenteral product are correct, EXCEPT:

a. there is minimal loss of activity in heat labile materials.

b. the liquid must be frozen to below the eutectic temperature.

c. the solute usually forms an amorphous glass.

*d. the eutectic temperature is the freezing point of the drug

solution.

e. water is removed from the frozen mixture by sublimation.

17 In cardiovascular physiology, LaPlace's law relates the tension

Page 68: Que & Ans_take Away

achieved in a blood vessel wall to:

I the length of the vessel.

*II the radius of vessel.

*III the intraluminal pressure.

a. I only

b. III only

c. I and II only

*d. II and III only

e. I, II and III

18 Concentration of a drug in breast milk exceeds that in plasma, if

the drug:

*a. is basic.

b. is protein bound.

c. is acidic but not protein bound.

d. has a small volume of distribution.

e. has a large volume of distribution.

19 Mercapturic acid derivatives in phase II metabolism can result from

reactions of:

*a. glutathione conjugates.

b. glucuronide conjugates.

c. glycine conjugates.

d. glutamate conjugates.

e. sulfate conjugates.

20 Tetrodotoxin is a selective inhibitor of the:

a. calcium ion channel.

*b. sodium ion channel.

c. potassium ion channel.

d. chloride ion channel.

e. nicotinic ion channel.

21 Atropine is the primary example of an important class of muscarinic

antagonists which are esters of tropic acid and which contain:

a. aniline and piperidine rings.

b. aniline and piperazine rings.

Page 69: Que & Ans_take Away

c. aniline and pyrrolidine rings.

d. pyrrolidine and piperazine rings.

*e. pyrrolidine and piperidine rings.

22 Carbon monoxide binds to:

*I hemoglobin.

*II myoglobin.

*III cytochrome oxidase.

a. I only

b. III only

c. I and II only

d. II and III only

*e. I, II and III

23 Antioxidants, which mechanistically act as reducing agents,

include:

*I ascorbic acid.

*II sodium bisulfite.

III citric acid.

a. I only

b. III only

*c. I and II only

d. II and III only

e. I, II and III

24 Diazepam Injection U.S.P

Diazepam 5 mg/ml

Ethanol 10%

Propylene glycol 40%

Benzyl alcohol 1.5%

Water for Injection qs 100%

In the formulation given above, the following ingredient(s) function(s)

as a cosolvent(s):

*I ethanol.

*II propylene glycol.

III benzyl alcohol.

Page 70: Que & Ans_take Away

a. I only

b. III only

*c. I and II only

d. II and III only

e. I, II and III

25 Both isoxazole and oxazole rings are found as substituents in some

bacteriostatic sulfanilamides. They are present in these structures

specifically because of their:

a. sizes and shapes.

b. tautomeric activities.

c. enzymatic activities.

d. electron-donating effects.

*e. electron-withdrawing effects.

26 The Michaelis-Menten equation will appear first order:

*a. when the substrate concentration is much smaller than Km.

b. when Km is much smaller than the substrate concentration.

c. when Vmax is much smaller than Km.

d. when Vmax is much larger than Km.

e. when Km approaches Vmax.

27 With respect to bio*****alence, the parameter “Cmax” is:

a. affected by the extent of absorption only.

b. affected by the rate of absorption only.

c. affected by neither rate nor extent of absorption.

*d. affected by both rate and extent of absorption.

e. the only significant parameter.

28

Which of the hydroxyls in the compound above is a tertiary alcohol?

a. Hydroxyl 1

*b. Hydroxyl 2

c. Hydroxyl 3

d. Hydroxyl 4

e. Hydroxyls 1 and 2

29

Page 71: Que & Ans_take Away

The above structures are related to one another as:

a. bioisosteres.

b. enantiomers.

*c. homologs.

d. rotamers.

e. positional (structural) isomers.

30 Five subjects given a single intravenous dose of a drug have the

following elimination half-lives: 3, 9, 6, 5 and 4 h (hours). The mean

half-life is:

a. 4.0 h.

b. 5.0 h.

*c. 5.4 h.

d. 5.8 h.

e. 6.0 h.

31 Reasons for using coatings on tablets include all of the following,

EXCEPT:

a. to mask the taste of the drug.

b. to mask the odor of the drug.

c. to improve the appearance of the tablet.

*d. to increase the drug's release rate.

e. to protect the drug from stomach acid.

32 Which of the following factors would be important in determining

the concentration of drug that would be reached in the cerebrospinal

fluid for treatment of meningitis?

*I Oil/water partition coefficient

*II Binding to plasma protein

*III pKa of the drug

a. I only

b. III only

c. I and II only

d. II and III only

*e. I, II and III

33 Which of the following terms indicates a loss of moisture?

Page 72: Que & Ans_take Away

a. Deliquescence

*b. Efflorescence

c. Hygroscopicity

d. Polymorphism

e. Condensation

34 Physicochemical factors affecting the rate and extent of

percutaneous drug absorption include:

*I the partition coefficient of the drug molecule.

*II the presence of surfactants in the formulation.

*III the type of ointment base.

a. I only

b. III only

c. I and II only

d. II and III only

*e. I, II and III

35 Which of the following statements is FALSE regarding tablet

formulation?

a. Diluents are fillers to add bulk to the tablet.

*b. Lubricants help the patient to swallow the tablet more easily.

c. Binding agents may be added dry or in solution.

d. Disintegrants draw water into the tablet causing it to burst.

e. Glidants promote the flow of materials during compression.

36 Structure-activity studies of cholinergic drugs indicate that:

I maximum activity is achieved when the quaternary nitrogen is three

carbons removed from the ester oxo group.

*II structures which display a quasi-ring conformation exhibit

muscarinic activity.

*III one methyl group of the trimethylammonium group can be replaced by

an ethyl group with only slight loss of activity.

a. I only

b. III only

c. I and II only

*d. II and III only

Page 73: Que & Ans_take Away

e. I, II and III

37 Which of the following is an azo dye?

a. Hexachlorophene

b. Nitrofurantoin

c. Methenamine

*d. Phenazopyridine

e. Nalidixic acid

38 If the total body clearance of a drug is 200 mL/min in a normal

healthy adult and the renal clearance is 10 mL/min then one may assume

that:

*a. the drug is extensively metabolized.

b. greater than normal drug accumulation would occur in patients with

moderate renal failure.

c. entero-hepatic recycling is significant.

d. the drug is not bound to plasma proteins.

e. the drug is concentrated in adipose tissue.

39 Approximately 50% of dicloxacillin is excreted unchanged in the

urine. If the normal dosage schedule for dicloxacillin is 125 mg q6h, a

patient with renal function 20% of normal should receive:

a. 25 mg q6h.

b. 31.25 mg q6h.

c. 62.5 mg q6h.

*d. 75 mg q6h.

e. 125 mg q12h.

40 Select the drug whose major metabolites have therapeutically

significant pharmacological activity:

a. oxazepam.

b. nitrazepam.

*c. diazepam.

d. triazolam.

e. clonazepam.

41 The major pathway for the biotransformation of the following

compound is by hydrolysis. Identify the site which would be most susceptible

Page 74: Que & Ans_take Away

to hydrolysis.

a. A

b. B

*c. C

d. D

e. E

42 Biceps muscles are part of:

*I the arm.

*II the thigh.

III the rib cage.

a. I only

b. III only

*c. I and II only

d. II and III only

e. I, II and III

43 The mechanism of antipsychotic effect of phenothiazines is thought

to be due to:

a. improvement of cholinergic transmission.

b. blockade of catecholamine re-uptake.

*c. blockade of dopamine receptors.

d. blockade of enkephalinergic neurons.

e. prolactin release.

44 Mechanisms of drug interactions with acetylsalicylic acid (ASA)

include:

a. induction of microsomal enzymes.

b. inhibition of microsomal enzymes.

c. inhibition of liposomal enzymes.

*d. displacement of other drugs bound to serum albumin.

e. enhanced platelet aggregation.

45 All of the following drugs will increase the heart rate directly or

by reflex when injected intravenously into a young healthy adult,

EXCEPT:

a. isoproterenol.

Page 75: Que & Ans_take Away

b. histamine.

c. phentolamine.

*d. phenylephrine.

e. atropine.

46 Which of the following diuretics is used to block the Na±H+

exchange system of the renal tubule?

a. Furosemide

b. Hydrochlorothiazide

c. Spironolactone

*d. Acetazolamide

e. Amiloride

47 Atropine poisoning can be recognized by all of the following signs

or symptoms, EXCEPT:

a. dry skin.

b. flushed appearance.

c. delirium and restlessness.

d. mydriasis.

*e. diarrhea.

48 The organophosphates commonly found in insecticides are thought to

act by which of the following mechanisms?

a. Combining with acetylcholine

b. Potentiating the action of acetylcholinesterase

*c. Forming a very stable complex with acetylcholinesterase

d. Reacting at the cholinergic receptor

e. Preventing the release of acetylcholine from the nerve ending

49 The skeletal structure of the hipbone includes the:

*I ilium.

*II ischium.

*III pubis.

a. I only

b. III only

c. I and II only

d. II and III only

Page 76: Que & Ans_take Away

*e. I, II and III

50 Pernicious anemia is:

a. due to dietary deficiency of vitamin B12.

b. prevented by oral administration of a vitamin B12 supplement.

c. treated by parenteral administration of folic acid.

*d. treated by parenteral administration of vitamin B12.

e. caused by dietary iron deficiency.

51 The optic disk is also called the:

*a. blind spot.

b. cornea.

c. iris.

d. pupil.

e. macula lutea.

52 Acarbose smooths and lowers:

a. postprandial blood glucose levels by blocking absorption of

carbohydrates.

*b. postprandial blood glucose levels by delaying absorption of

carbohydrates.

c. preprandial blood glucose levels by blocking absorption of

carbohydrates.

d. preprandial blood glucose levels by delaying absorption of

carbohydrates.

e. preprandial and postprandial blood glucose levels by blocking

absorption of carbohydrates.

53 Autoimmune disorders include which of the following?

*I Graves' disease

*II Systemic Lupus Erythematosus

III Osteoarthritis

a. I only

b. III only

*c. I and II only

d. II and III only

e. I, II and III

Page 77: Que & Ans_take Away

54 Acute hemorrhagic gastritis is commonly seen with:

*a. chronic alcoholism.

b. H. pylori bacterial infections.

c. carcinoma of the stomach.

d. carcinoma of the colon.

e. Travellers' diarrhea.

55 True statements regarding the use of monoclonal antibodies include

which of the following?

*I Flu-like symptoms commonly occur at the start of therapy.

*II T cells are blocked from initiating the rejection process.

III Orthoclone is a human immunoglobulin product.

a. I only

b. III only

*c. I and II only

d. II and III only

e. I, II and III

Questions 56 to 57 inclusive refer to the following:

KJ, a 23 year-old female, is to receive lithium therapy with a regimen

of lithium carbonate 600 mg q8h. The total body clearance of lithium is

0.44 mL/s (1.6 L/h) and the biological half-life is 18 h. The

molecular weight of lithium carbonate (Li2CO3) is 74.

56 The number of millimoles of lithium (Li+1) that KJ will receive

with each dose is:

a. 8

*b. 16

c. 24

d. 32

e. 48

57 The number of milli-*****alents of lithium that KJ will receive

with each dose is:

a. 8

*b. 16

c. 24

Page 78: Que & Ans_take Away

d. 32

e. 48

58 The fraction of bound receptors in a well-defined drug-receptor

binding system is determined by:

*I the equilibrium drug-receptor dissociation constant.

*II the free drug concentration.

III the total receptor concentration.

a. I only

b. III only

*c. I and II only

d. II and III only

e. I, II and III

59 In some parenteral formulations, sodium metabisulphite is included

as:

*a. an antioxidant.

b. a chelator.

c. a solubilizer.

d. a cosolvent.

e. an emulsifier.

60 Potassium-sparing diuretics:

I exert their effect in the proximal tubule.

*II may cause intracellular alkalosis.

*III include the aldosterone antagonists.

a. I only

b. III only

c. I and II only

*d. II and III only

e. I, II and III

61 Dorzolamide:

I has a sulfonylurea structure.

*II has carbonic anhydrase inhibitor activity.

*III is useful in the treatment of glaucoma.

a. I only

Page 79: Que & Ans_take Away

b. III only

c. I and II only

*d. II and III only

e. I, II and III

62 True statements regarding transcription include which of the

following?

*I The enzyme responsible for initiating transcription is RNA

polymerase.

*II The genetic information contained in the nucleotide sequence of

mRNA is translated into a protein structure.

*III The information carried in the gene sequence is transferred to

RNA.

a. I only

b. III only

c. I and II only

d. II and III only

*e. I, II and III

63 Which of the following agents may cause pulmonary toxicity?

*I Amiodarone

*II Bleomycin

III Ramipril

a. I only

b. III only

*c. I and II only

d. II and III only

e. I, II and III

64 An angiotensin-converting enzyme (ACE) inhibitor for which both

renal elimination and metabolism are important in the elimination of the

drug and its active metabolites is:

a. enalapril.

*b. fosinopril.

c. lisinopril.

d. quinapril.

Page 80: Que & Ans_take Away

e. ramipril.

65 Which of the following agents is NOT associated with the toxic

effect of Stevens-Johnson syndrome?

a. Warfarin

b. Cotrimoxazole

*c. Amoxicillin

d. Phenobarbital

e. Lamotrigine

66 Once daily dosing of aminoglycosides is effective due to:

a. prolonged residence of the antibiotic in the body.

*b. post-antimicrobial effect.

c. enhanced tissue accumulation.

d. reduced renal clearance.

e. higher peak-trough differences.

67 The method of ionization of mass spectrometry which results in

well-established diagnostic fragmentation patterns that are useful in the

identification of compounds of unknown structure is called:

a. atmospheric pressure chemical ionization (APCI).

b. chemical ionization (CI).

*c. electron impact (EI) ionization.

d. electrospray ionization (ESI).

e. fast atom bombardment (FAB) ionization.

Pharmacy Practice

68 The ethical principle of veracity requires that:

a. we respect the rights of others to make choices.

*b. we act with honesty, without deception.

c. we avoid, remove or prevent harm.

d. we do good to patients.

e. we act with fairness.

69 MT, a pharmacy technician, has worked in a community pharmacy for 5

years. He is well-trained and welcomes new learning opportunities. His

manager, now wanting to delegate a new technical project to him,

should:

Page 81: Que & Ans_take Away

*I discuss suggested steps to accomplish the project.

*II negotiate a time schedule for completion of the project.

*III provide support and followup as required.

a. I only

b. III only

c. I and II only

d. II and III only

*e. I, II and III

70 Which of the following antacids has cathartic side-effects?

a. Calcium carbonate

b. Dihydroxyaluminum sodium carbonate

*c. Magnesium hydroxide

d. Calcium sulfate

e. Aluminum silicate

71 Side effects that may be expected to occur from the administration

of oral contraceptives include all of the following, EXCEPT:

a. fluid retention.

b. headache.

c. hypertension.

*d. constipation.

e. depression.

72 In deciding what drugs are appropriate for its formulary, the

hospital must consider a drug's efficacy, associated workload, and

acquisition cost. Several new antifungal IV drugs (drug A, drug B, drug C, and

drug D), all with equal efficacy, have recently become available.

Currently the hospital stocks drug E, which has been available for several

years. Data for the medications is as follows:

DrugCost/DayDosing FrequencyTreatment Duration

(days)A$2.50QID14B$2.25once daily14C$5.00BID7D$2.25BID7E$2 .25QID7The most appropriate choice

for the hospital is:

a. Drug A.

b. Drug B.

c. Drug C.

Page 82: Que & Ans_take Away

*d. Drug D.

e. Drug E.

73 When evaluating patients with angina for possible development of

nitrate tolerance, true statements include:

I tolerance is rarely associated with isosorbide dinitrate.

II tolerance does not occur in patients who are concurrently on

metoprolol.

*III tolerance is dependent upon the administration schedule of nitrate

medication.

a. I only

*b. III only

c. I and II only

d. II and III only

e. I, II and III

74 Which financial statement could be used to determine the total

value of prescription drug sales for a pharmacy during the course of a

year?

a. Balance sheet

b. Statement of investments

c. Statement of changes in financial position

*d. Income statement

e. Statement of equity

75 The Latin abbreviation for "before meals" is:

*a. ac.

b. aa.

c. pc.

d. ic.

e. cc.

76 Erythromycin:

*I is effective against gram-positive cocci.

*II is a macrolide antibiotic.

*III can increase gastrointestinal tract motility.

a. I only

Page 83: Que & Ans_take Away

b. III only

c. I and II only

d. II and III only

*e. I, II and III

77 Aerosol OT (AOT) is used in veterinary medicine as a laxative. If

250 g of AOT is dissolved in 1 000 mL of glycerin (density of glycerin

is 1.25 g/mL), the concentration of AOT in the solution is:

a. 12.5% w/w.

*b. 16.7% w/w.

c. 20.0% w/w.

d. 23.8% w/w.

e. 25.0% w/w.

78 Water used for reconstitution of an additive for a large volume

parenteral should be:

a. sterile water.

b. distilled water.

c. Bacteriostatic Water for Injection (United States Pharmacopeial

standard).

d. Water for Injection (United States Pharmacopeial standard).

*e. Sterile Water for Injection (United States Pharmacopeial standard).

79 The style of management in which the owner of a pharmacy emphasizes

the development of detailed policies and written procedures for

employees to observe at all times is referred to as:

a. autocratic.

*b. bureaucratic.

c. participative.

d. benevolent.

e. consultative.

80 The shock and airway edema of anaphylaxis are best treated with:

a. salbutamol.

b. diphenhydramine.

*c. adrenaline.

d. acetazolamide.

Page 84: Que & Ans_take Away

e. aminophylline.

81 Sulfinpyrazone is used as a(n):

a. analgesic agent.

b. antipyretic agent.

c. anti-inflammatory agent.

d. agent which increases renal tubular reabsorption of uric acid.

*e. uricosuric agent.

82 In the treatment of rheumatoid arthritis, the salicylates:

a. stop and often reverse the progressive joint involvement.

b. specifically reverse the cause of the disease.

*c. provide only analgesic and anti-inflammatory effects.

d. are effective because they are uricosuric agents.

e. are more effective when given with allopurinol.

83 Prednisone may produce all of the following effects, EXCEPT:

a. skeletal muscle weakness.

*b. hypoglycemia.

c. sodium retention.

d. peptic ulceration.

e. lowered resistance to infection.

84 In an adequately powered, randomized controlled trial conducted

over 2 years, the desired clinical outcome (i.e. prevention of a serious

cardiovascular event) with a new drug is achieved in 25% of the study

sample. In the patients who receive a placebo, only 15% obtain the same

clinical benefit. The relative risk reduction achieved with the new drug

over the study period is:

a. 10%.

b. 15%.

c. 25%.

*d. 40%.

e. 50%.

85 In an adequately powered, randomized controlled trial conducted

over 3 years, a specific serious side effect (i.e. reduction in

leukocytes) with conventional therapy is seen in 0.5% of the study sample. In

Page 85: Que & Ans_take Away

patients who receive a newly discovered drug, only 0.45% experience the

same side effect. Based on these results, the minimum number of patients

that would have to receive the new drug for 3 years to statistically

demonstrate the prevention of one episode of this side effect in at least

one patient is:

a. 15.

b. 20.

c. 150.

d. 200.

*e. 2000.

86 The pharmacist fills a prescription for sumatriptan 100 mg tablets

for a migraine patient. Appropriate information to provide to the

patient includes which of the following?

a. If the sumatriptan does not relieve the headache within four

hours, ergotamine may be used.

b. If no relief is achieved in two hours, sumatriptan may be

repeated.

*c. If the headache is relieved but another headache occurs eight hours

later, sumatriptan may be used for the second headache.

d. The maximum dosage in any 24 hour period is six tablets.

e. If relief is not achieved, no other medication can be used for 24

hours.

87 Auxiliary labels which should be used on cotrimoxazole suspension

include which of the following?

*I Shake well before using.

*II Take with plenty of fluids.

*III Exposure to sunlight may cause adverse reactions.

a. I only

b. III only

c. I and II only

d. II and III only

*e. I, II and III

88 A patient purchasing sublingual nitroglycerin tablets should be

Page 86: Que & Ans_take Away

told to store the medication:

a. in the refrigerator.

b. in a plastic vial with a childproof cap.

c. with the original cotton.

*d. in an amber glass bottle with a metal cap.

e. in a warm, dry place.

89 Factors that determine bio*****alence of two brands of a drug

include:

I the taste of the preparations.

II the physical appearance of the preparations.

*III pharmacokinetic parameters of the preparations.

a. I only

*b. III only

c. I and II only

d. II and III only

e. I, II and III

90

Rx Timolol 0.25% dropsMitte: 15 mLSig: gtt. i o.d. BIDOn the

prescription label the instructions to the patient should read:

a. apply one drop into both eyes twice daily.

b. instil one drop into the left eye twice daily.

*c. instil one drop into the right eye twice daily.

d. shake well and instil one drop into the left eye twice daily.

e. shake well and instil one drop into the right ear twice daily.

91 Following the ethical principle of nonmaleficence requires that

pharmacists:

a. respect the rights of patients to make choices.

b. do good to patients, placing the benefit of the patient above all

else.

*c. avoid, remove or prevent harm from people.

d. act with fairness, allowing people access to pharmacy care.

e. act with honesty, without deception.

92 Dopamine is useful in the treatment of cardiogenic shock because

Page 87: Que & Ans_take Away

it:

*I selectively dilates renal and mesenteric vascular beds.

II does not induce peripheral vasoconstriction.

III decreases the force of myocardial contraction.

*a. I only

b. III only

c. I and II only

d. II and III only

e. I, II and III

93 The following prescription is received in a community pharmacy:

Losec® 20 mg

S: Take 1 tab BID

M: 2 weeks

Which of the following is correct labelling for this prescription?

a. Take one tablet twice weekly (28 tablets).

b. Take one tablet twice weekly (4 tablets).

c. Take two tablets twice daily (56 tablets).

*d. Take one tablet twice daily (28 tablets).

e. Take one tablet twice daily (14 tablets)

The next section includes an EXAMPLE OF THE CASE SCENARIO FORMAT, in

which there are two or more questions in sequence, that are related to

the patient information provided in the (bolded) stem shown at the top of

the case.

When choosing your response to questions in the case scenario format,

you must base your choice on the patient-specific information presented

in the case.

Questions 94 to 96 inclusive refer to the following:

SM is a 34 year old female who, while vacationing in Mexico, began

prophylactic treatment for Travellers' Diarrhea. Shortly thereafter she

complained of a feeling of fullness in her ears, black stools and a black

tongue. SM’s previous history includes an allergy to sulfonamides.

94 Which of the following drugs could be the cause of SM's complaints?

*a. Bismuth subsalicylate

Page 88: Que & Ans_take Away

b. Cotrimoxazole

c. Doxycycline

d. Amoxicillin

e. Loperamide

95 Organisms commonly implicated in the cause of Traveller's Diarrhea

include:

I Bacteroides fragilis.

*II Escherichia coli.

*III Shigella sp.

a. I only

b. III only

c. I and II only

*d. II and III only

e. I, II and III

96 Drugs shown to be effective in the prevention of Travellers'

Diarrhea, which might have been prescribed for SM include:

*I doxycycline.

II cotrimoxazole.

III amoxicillin.

*a. I only

b. III only

c. I and II only

d. II and III only

e. I, II and III

Top of Form

Delete Reply Forward

Bottom of Form

Page 89: Que & Ans_take Away

(4)

BONES AND JOINT COMPLICATIONS

1- Osteoporosis is a disease characterized by loss of bone mass and may be aggravated by:

I-Corticosteroids

II-Menopausa

III-Grave’s disease

a) I only

b) III only

c) I and II only

d) II and III only

e) All are correct

2- A patients with allergy to Sulphonamides (i.e.: Sulphametoxazole) can take all of the following medications, EXCEPT:

a) Penicillin

b) Lidocaine

c) Celecoxib

d) Clindamycin

e) Tetracycline

3- All of the following are examples of DMRA-Disease Modifying Antirheumatic Agents, EXCEPT

a) Methothrexate

b) Sulfasalazine

c) Organic Gold Compounds

d) Sulfinpyrazone

e) Penicillamine

4- Calcitonin is mainly classified as hormone liberated by thyroid gland. Which stimulate the release of calcitonin?

a) Hypercalcemia

b) Hypocalcemia

c) Hypophosphatemia

d) Hyperphosphatemia

e) High concentration of Vit. D

Page 90: Que & Ans_take Away

5- Which of the following statements is/are right regarding CALCITONIN

I- It secretion is normally stimulated during feeding, particularly of high calcium meals

II- Prevents hypercalcemia by inhibiting the transport of Ca intro extra cellular fluid and by increasing the renal clearance of calcium

III- It is a vitamin D derivative

a) I only

b) III only

c) I and II only

d) II and III only

e) All are correct

6- The level of which drug should be adjusted during treatment with allopurinol

I- Mercapturic acid

II- Theophillin

III- Anticoagulants

a) I only

b) III only

c) I and II only

d) II and III only

e) All are correct

7- Uricosuric agents are used in gout treatment by inhibiting the uric acid re-absorption in the proximal tubules, the best example of a uricosuric agent is:

a) Allopurinol

b) sulphinpyrazone

c) Indomethacin

d) Penicillamine

e) Methotrexate

8- An oral chelating agent used in the treatment of rheumatoid arthritis is:

a) Allopurinol

b) Sulphinpyrazone

Page 91: Que & Ans_take Away

c) Indomethacin

d) Penicillamine

e) Methotrexate

9- In the treatment of rheumatoid arthritis the salicylates:

a) Stop and often reverse the progressive joint involvement

b) Specifically reverse the cause of the disease

c) Provide only analgesic effect

d) Are effective because they are uricosuric agents

e) Are more effective when given with allopurinol

10- Importance of Vitamin D in osteoporosis:

I- Increase the absorption of calcium from the small intestine

II- The best vitamin D analog to treat osteoporosis is calcitriol

III- Antacids containing aluminum and magnesium may interfere with proper activity of vitamin D.

a) I only

b) III only

c) I and II only

d) II and III only

e) All are correct

11- Drug that inhibits xanthine oxidase and affect the metabolism of 6-mercaptopurine:

a) Sulfinpyrazone

b) Allopurinol

c) Penicillamine

d) Purinethol

e) Celecoxib

12- Pannus is an inflammatory exudates characterized by:

I- Synovial cell proliferation inside of a joint

II- Normally occurs in patients with rheumatoid arthritis

III- An immunologic complication

a) I only

Page 92: Que & Ans_take Away

b) III only

c) I and II only

d) II and III only

e) All are correct

13- Intra-articular means:

a) In the joint space

b) In the tissues space

c) In the spinal fluid

d) In the spinal column

e) In the dermis

14- Colchicines, a widely drug used in gout treatment, can give false results of:

a) Glucose

b) Uric acids

c) Urates

d) Ketones

e) RBCs

15- Colchicine is used as anti-inflammatory agent in the acute treatment of:

a) Osteoporosis

b) Rheumatoid arthritis

c) Osteomyelites

d) Gout

e) Pressure atrophy

16- The use of indomethacin in gout treatment:

a) Inhibit the acute gout arthritis inflammation by inhibition of prostaglandin formation

b) Inhibits tubulin synthesis

c) Accelerate renal excretion of uric acid

d) Inhibit uric acid synthesis

e) Inhibit the reabsorption of uric acid by the kidney

Page 93: Que & Ans_take Away

17- Allopurinol, a xanthine oxidase inhibitor, is a derivative of:

a) Uric acid

b) Purine

c) Pyridine

d) Pteridine

e) Urate salts

18- Goals of gout treatment may include:

I- Reduce inflammation during acute attacks

II- Accelerate renal excretion of uric acid

III- Reduce the conversion of purine to uric acid

a) I only

b) III only

c) I and II only

d) II and III only

e) All are correct

19- Characteristics of GOUT, a group of disorders of purine and pyrimidine metabolism may include:

I- Hereditary metabolite disease marked by inflammation of the joints

II- The joint inflammation is caused by precipitation of uric acid crystals

III- It is a type osteoporosis

a) I only

b) III only

c) I and II only

d) II and III only

e) All are correct

20- Allopurinol, xanthine oxidase inhibitor, used in gout treatment may produce all the following side effects, EXCEPT:

a) Increase in protrombin time

b) Hypersensitivity rashes

Page 94: Que & Ans_take Away

c) GIT intolerance

d) Peripheral neuritis

e) Necrotizing vasculitis

21- Use of estrogen in osteoporosis:

I- Estrogen enhance calcium retention and retard bone loss

II- Estrogen is useful in treat osteoporosis associated to menopause

III- Estrogen is not effective at increasing bone mass that has already been lost.

a) I only

b) III only

c) I and II only

d) II and III only

e) All are correct

22- Osteoporosis is mainly characterized by:

a) Loss of joint movements

b) Loss of bone mass

c) Imbalance of uric acid

d) Hormonal imbalance

e) All are correct

23- Antimalarial drug used in the treatment of rheumatoid arthritis as DMRA-Disease-Modifying Antirheumatic Drug

a) Chloroquine

b) Quinine

c) Sulfasalazine

d) Hydroxycloroquine

e) Primaquine

24- Which is the correct start dose of methothrexate in the treatment of rheumatoid arthritis?

a) 7.5mg once a week

b) 15mg once a week

c) 7.5mg daily

Page 95: Que & Ans_take Away

d) 15mg daily

e) 22mg once a week

25- Correct administration of Penicillamine in the treatment of rheumatois arthritis include:

a) With meals

b) In emptying stomach

c) With plenty of water

d) With orange juice

e) Before bedtime

26- Which of the following agents is the best choice of treatment for an asthma patient with rheumatoid arthritis who is considered to have aspirin sensitivity?

a) Ibuprofen

b) Acetaminophen

c) Gold therapy

d) Azathioprine

e) Cyclophosphamide

27- Diflunisal is most likely to be prescribed for the treatment of:

a) Wilson’s disease

b) Rheumatoid arthritis

c) Cysteinuria

d) Psoriasis

e) Hansen’s disease

1- E

Comments: Osteoporosis is a disease characterized by the loss of bone mass. Substances or conditions that alter the metabolism of bones may predispose or aggravated osteoporosis. Example: Corticosteroids, menopause and Grave’s disease.

2- C

Comments: There is a cross-sensitivity drug interaction between sulphonamides and celecoxib

3- D

Comments: Sulphynpyrazone is an uricosuric acid agent used in the treatment of gout.

Page 96: Que & Ans_take Away

4- A

Comments: Calcitonin is a hormone elaborated by the thyroid gland in response to hypercalcemia.

5- C

Comments: Calcitonin is not a vitamin D derivative. It is a hormone elaborated by the thyroid gland in response to hypercalcemia, lowering plasma calcium and phosphate levels.

6- E

Comments: Allopurinol may inhibit enzymatic metabolism of anticoagulants, leading to potentiation of the anticoagulant effect. Mercapturic acid dosage should be reduced with concomitant use with allopurinol because it induced inhibition of xanthine oxidase decreasing metabolism of mercapturic acid and potentiating therapeutic and toxic effects. Allopurinol decrease theophyllin clearance resulting in increased serum theophyllin concentrations

7- B

Comments: Allopurinol and indomethacin are used in gout treatment however not as uricosuric agent but as xanthine oxidase inhibitor and NSAID respectively. Methotrexate is used to treat rheumatoid arthritis as DMRA agent.

8- D

Comments: Allopurinol and sulphynphyrazone are only used in gout treatment. Indomethacin can be used in rheumatoid arthritis however as NSAID and Methotrexate is used as DMRA agent.

9- C

Comments: Salicylates in the treatment of rheumatoid arthritis provides only analgesic effect by reducing prostaglandin synthesis.

10- E

Comments: Osteoporosis is characterized by the loss of bone mass. Vitamin D supplements have been used in the treatment of osteoporosis because vitamin D increases the absorption of calcium that is essential to reduce bone loss. Antacids interact with vitamin D in many ways, magnesium containing antacids may lead to hypermagnesemia and aluminum containing antacids have its levels increased leading to aluminum bone toxicity when currently used with vitamin D.

11- B

Comments: Allopurinol inhibit xanthine oxidase enzyme therefore the conversion of 6-mercaptopurine to uric acid inhibiting the uric acid synthesis-formation

Page 97: Que & Ans_take Away

12- C

Comments: Pannus is an inflammatory exudates overlying the lining layer of synovial cells on the inside of a joint, usually occurring in patients with rheumatoid arthritis or related articular rheumatisms, and sometimes resulting in fibrous ankylosis of the joints.

13- A

Comments: Intra-articular is normally referred to a kind of injection given within a joint.

14- E

Comments: Colchicine may cause false-positive test results of red blood cells-RBC and hemoglobin in urine. It may also cause decrease in platelet count.

15- D

Comments: Cochicine is only used in gout treatment. It has no effect on uric acid therefore it is used just in the acute treatment of gout by inhibiting the migration of leukocytes, phagocytes and leukotrienes B4 to the affected joint.

16- A

Comments: Indomethacin is a potent NSAID that inhibits the acute gout arthritis inflammation by inhibition of prostaglandin formation and inhibition of crystals phagocytes by macrophages.

17- B

Comments: Allopurinol is a purine derivative working as xanthine oxidase enzyme inhibitor responsible for the conversion of 6-mercaptopurine to uric acid.

18- E

Comments: The gout disease is normally controlled by three different mechanisms: 1- Control the inflammatory response by inhibition of leukocytes, macrophages and leukotrienes migration to the affected joint. 2- Increase the renal clearance of uric acid by inhibiting the reabsorption of uric acid by the kidney and 3- Reduce the conversion of 6-mercaptopurine to uric acid by inhibition of the enzyme xanthine oxidase.

19- C

Comments: Gout is a group of disorders of purine and pyrimidine metabolism. It is manifested by hyperuricemia, recurrent characteristic arthritis induced by deposit of crystals in and around the joint of the extremities, which may lead to destruction and severe crippling of joints.

20- A

Page 98: Que & Ans_take Away

Comments: Main side effects of allopurinol therapy include: Steven-Johnson’s hypersensitivity rashes reactions, gastro-intestinal intolerance, peripheral neuritis and necrotizing vasculitis.

21- E

Comments: Estrogen is widely used to prevent bone loss after menopause. Estrogen can enhance calcium retention and retard bone loss but does not increase bone mass that has already been lost.

22- B

Comments: Osteoporosis is mainly characterized by the loss of bone mass. It is the most common of the metabolic bone diseases and is an important cause of morbidity in elderly.

23- D

Comments: Cloroquine and primaquine are normally used together only in the treatment and prevention of malaria. Quinine and hydroxycloroquine are antimalarial agents and alternatively used in prophylaxis and treatment of muscle cramps leg and rheumatoid arthritis respectively.

24- A

Comments: Methotrexate is a Disease-Modifying rheumatoid arthritis agent normally used orally, 7.5mg once a week or 2.5mg every 12hours for 3 doses given as a course once weekly, increased 15mg per week after 6 weeks, if necessary.

25- B

Comments: In the treatment of rheumatoid arthritis penicillamine should be administrated in an empty stomach. Improvement of the condition may take up to 2 to 3 months of therapy

26- C

Comments: Gould therapy would be the best choice for this particular patient because the other drugs may have cross-sensitivity with aspirin and/or induce brochospasm.

27- B

Comments: Diflunisal is a NSAID used for both acute and chronic anti-inflammatory treatment of rheumatoid arthritis.

Page 99: Que & Ans_take Away

Share

(5)

CARDIOVASCULAR SYSTEM

1. Antidiuretic characterized for the decrease of Na+,K+ and Cl-.

a. Acetazolamide

b. spirinolactone

c. hydrochlorthiazide

d. furosemide

e. triamterene

2. Antithrombolytic agents are contraindicated in:

a. Myocardial Infarction

b. bleeding from the GIT

c. hyperchloremic patient

d.

3.Patient with renal failure and normal liver function is showing sensitibility to the drug, this means that:

a. high accumulation of metabolite

b. high concentration of drug protein

c. high VD

d. high concentration of drug in systemic circulation

e. high accumulation of drugs in tissues

4. Normal osmotic blood pressure in a healthy adult is:

a. 280 mosm/L

b. 280 Osm/L

c. 280 mOsm/mL

d.280 Osm/mL

e. 280 mOsm/100 mL

Page 100: Que & Ans_take Away

5. Which increase water permeability at collecting tubules?

A. ADH

B. Hydrochlorthiazide

C. Aldosterone

D. Triamterene

E. Spirinolactone

6. Digoxin is cardiac digitalis widely used in CV which toxicity in increased by:

I. Erythromycin

II. Quinidine

III. Cholestyramine

7. Acetazolamide, a carbonic anhydrase diuretic and hydrochlorthiazide, a thiazide diuretic have an identical molecular group very important in their structure known as:

a. sulphonamide group

b. pyrimidine structure

c. purine group

d. pyrrolidine

8. A man with blood group AB can receive blood from:

a. A

b. B

c. AB

d. O

e. all of the above

9.When would you advise patient that he may suffer from hypertension and he may need to a doctor.

I. His blood monitor shows BD 100/160

II. The pharmacy monitor shows BP 100/160

III. His meter shows 80/120

10. During absolute refractory period the myocardial fiber.

Page 101: Que & Ans_take Away

a. receive the strongest stimulation

b. it is relaxed but is able to respond to stimulus

c. responds to the most sensible stimulus

d. do not respond to any stimulus

e. none of the above

11. Abnormal persistent dilation of blood vessel is known as:

a. embolism

b. aneurism

c. emphysema

d. starvation

e. artherosclerosis

12. Correct statements regarding embolism include:

I. Abnormal dilation of blood vessel

II. Obstruction of a blood vessel by a mass transported in the circulation

III. An embolism can be a clot or a foreign material transported in the circulation

13. Which of the following statements regarding Heparin is right:

I. It is a mucopolysaccharide, organic acid present naturally in many tissues especiallyin lungs and liver

II. It has anticoagulant properties that inhibit the conversion of prothrombin to thrombin and fibrinogen to fibrin

III. It has antithrombin III as its co-factor

14. Heparin and warfarin are very popular anticoagulant agents. What is wrong regarding heparin and warfarin

a. both have anticoagulant properties

b. warfarin has activity in vivo & in vitro

c. Heparin has activity only in vivo

d. Heparin has activity in vivo & in vitro

e. all are correct

15. Which of the following calcium channel blockers cause bradycardia:

I. Nifedipine

II. Verapamil

Page 102: Que & Ans_take Away

III. Diltaizem

16. Important characteristics regarding the blood formed elements may include:

I. RBC- Red blood cells are originated from erythrocytes and have a half-life of 120 days (4 months)

II. Platelets are involved in blood clot and have half-life of 7 days

III. Platelets are originated from myoleoriocytes

17. Warfarin, an anticoagulant agent has its effects increased by:

I. Deficiency of Vit.K

II. Adjunct therapy with heparin

III. Adjunct therapy with salicylates

18. All are drugs that decrease LDL except:

a. clofibrate

b. genfibrozil

c. cimetidine

d. simvastatin

e. pravastatin

19. A patient with deep vein thrombosis can be treated by:

I. Streptokinase

II.Heparin

III. warfarin

20. Alpha blockers are readily used in CV complications. One of the biggest cautions in using ∂ -blockers is the development of:

a. tachycardia

b. bradycardia

c. orthostatic hypotension

d. hypotension

e. increase in cardiac output

21. All of the following are ACE inhibitors except:

a. enalapril

Page 103: Que & Ans_take Away

b. captopril

c. ramipril

d. lisinopril

e. Losartan

22. During inspiration the lowest blood pressure in supine position is:

a. vena cava

b. internal carotid artery

c. aorta artery

d. sciatic artery

e. pulmonary artery

23.Which of the following theories relates the tension, radius of vessel and intraluminal pressure achieved in a blood vessel wall:

a. Fick’s Law

b. Ahhrenius

c. La Place’s Law

d. Newtonian Law

e. Henderson Law

24. The first ******** where blood is present in case of left ventricular failure

a. liver

b. lungs

c. heart

d. kidney

e. heart

25.The blood weighs 7 to 8% of the total body weight of a healthy adult.How much of these percentage are represented approximately in liter of blood

a. 4-6 L

b. 7-10 L

c. 7-8 L

d. 10 L

e. 15 L

Page 104: Que & Ans_take Away

26. Nitrates are mainly used in angina treatment. True statement regarding nitrates:

I. Nitrates are esters of nitrous and nitric acid

II. Nitrates are prodrugs and must be denitrated to exert their therapeutic effect.

III. Nitrates relaxes vascular smooth muscle and in addition relax bronchial, biliary, gastrointestinal and uterine smooth muscles.

27. Which of the following diuretics is used to block Na/H exchange system of the remal tubule.

a. Indapamide

b. Ouabain

c. spironalactone

d. acetazolamide

e. amiloride

28. Example of diuretics that ↑ levels of Cl-, causing hyperchloremic is:

a. acetazolamide

b. amiloride

c. furosemide

d. ouabain

e. hydrochlorthiazide

29. Dobutamine is a direct acting inotropic agent parenterally used in cardiac emergencies and characterized by:

I. Increased myocardial contractility, very useful in myocardial infarction

II. Increased cardiac output

III. Increased glomerular filtration, renal blood flow and sodium excretion

30. Spironolactone is an aldosterone antagonist diuretic that competes with aldosterone at:

a. proximal tubule

b. early distal

c. early collecting tubule

d. collecting tubule

e. end of collecting tubule

31. Examples of drug affecting platelet aggregation may include:

I. Ticlopidine

Page 105: Que & Ans_take Away

II. Clopidogrel

III. Aspirin

32. What kind of arrhythmia can be caused by an increase in temperature.

A . Ventricular premature depolarization

B . atrial fibrillation

C . paroxysmal arrhythmia

D . Ventricular tachycardia

E . none of the above

33. Prazosin is a sympathetic agent used in hypotension acting as:

A . ∂ -adrenergic blocker

B . β adrenergic blocker

C .

d.

e . ∂ & β adrenergic blocker

34. All are side effect of hydrochlorthiazide except:

a. hypokalemia

b. hypoglycemia

c. hyponatremia

d. hypercalcemia

e. hyperuricemia

35. CHF is a cardiac disorder which:

I. Result from any cardiac disorder that impairs the ability of the ventricle to deliver adequately quantity of blood to the metabolizing tissues.

II. result in poor pump function

III. β blockers are the 1st treatment for CHF

36. Drugs that may affect the normal activity if the warfarin include:

a. Ibuprofen

Page 106: Que & Ans_take Away

b. Aspirin

c. Vit K

d. acetaminophen

e. all are correct

37. Glycosides are widely used in the treatment of CHF. Glycosides molecular structure consist of steroidal nucleon and:

a. sugars

b. polypeptides

c. carbohydrates

d. proteins

e.

38. Stenosis is :

a. dilatation of blood vessels

b. increase in cardiac output

c. accumulation of cholesterol in blood vessel

d. narrowing of blood vessel

e. high pressure in blood vessel

39. Cardiac output at rest

a. lungs

b. liver

c. kidney

d. muscle

e. fat

40. Calcium channel blockers that may cause myocardial infarction as side effect include:

I. diltiazem

II. verapamil

III. nifedipine

41. Bradycardia and orthosthatic hypotension are characteristic side effect of:

I. guanebenz

Page 107: Que & Ans_take Away

II. hydralazine

III. vasodilation

42. An important advantage of using dopamine in cardiac shock may include:

a. It will not cross blood brain barrier cause of CNS effect

b. it has no effect on ∂ & β receptor

c. It produce dose dependent increasing cardiac output and renal perfusion

d. it will not increase blood pressure

e. it can be given orally

43. Which of the following diuretics may show a uricosoric activity

a. spirinolactone

b. furosemide

c. chlortalidone

d. triamterene

e. chlorothiazide

44. Ancrod, are antithrombolytic agent that is characterized by:

I. It is derivated from snake venom

II. It depletes fibrinogen

III. It is an alternative to heparin

45. The use of sildenafil together with atenolol may result in:

I. tremor

II. Hypotension

III. Visual disturbances

46. a diabetic patient suffering from hypertension is having enalapril. However he is complaining from severe dry cough that we can associate as a side effect of the enalapril. Which would be the best choice for changing this medication.

a. Ramipril

b. losartan

c. amlodipine

Page 108: Que & Ans_take Away

d. indapamide

e. captopril

47. Drug of choice when treating hypertension in a diabetic patient may include:

a. ACE inhibitor

b. β blocker

c. Calcium Channel blocker

d. digitalis

e. nitrates

48. Angina, a cardiovascular complication characterized by oxygen insufficiency may be aggravated by all of the following except:

a. exercise

b. hot places

c. cold places

d. anger

e. overweight

49. Drugs used in the treatment of hypertension associated with renal failure may include:

I. Hydrochlorthiazide

II. Furosemide

III. Atenolol

50. Digitalis is contraindicated in the treatment of:

a. Myocardial Infarction

b. Coronary Heart Disease

c. congestive Heart Failure

d. Arrythmia

e. venous congestion

51. Systemic Lupus or erythomatus lupulus may be a side effect of:

a. Propanolol

b. Digoxin

c. cyclophosphamide

d. hydralazine

e. hydroxyzine

Page 109: Que & Ans_take Away

52. Which best describe the properties of labetalol

a. β blocker

b. ∂ blocker

c. ∂ & β 1 blocker

d. β non selective & ∂ blocker

e. β-2 blocker

53. Correct statement regarding blood pressure may include:

I. Systolic pressure is the highest arterial pressure during cardiac cycle

II. Diastolic pressure is the lowest arterial pressure during cardiac cycle

III. Diastolic pressure is crucial in verifying if a person suffers of high blood pressure

54. Cholesterol

I. VLDL (very low density lipoproteins) are formed in the liver

II. LDL (Low density Lipoprotein) are formed from VLDL and carry LDL cholesterol that is the bad cholesterol

III. HDL (high density lipoproteins) are

(6)

Evaluation exam July 2004

Q1.Which is the most commonly user lubricant?

Ans .mg stereate

Q2.alpha pheto protein is elevated in which condition?

Ans. Liver chrosis

Q3.which improve the flow properties of powder?

Ans.glidant

Page 110: Que & Ans_take Away

Q4.what is the mode of action of orlistat?

Q5.what is the absolute contraindication of antithrombolytic drugs? K type

I.GI BLEEDING

II.AGE OVER 65

III.ISCHEMIC ATTACK

ANS.I ONLY

Q6.which drug can be given in the mild to moderate hypertesion whith chf?

a.beta blocket

b.ca cahnnel blocker

c.hydrochlorothiazide

ans. B

Q7. what is the mode of action of fiestride?

Ans.

Q8. which stucture resemble to gaba?

Ans

Q9.what is the English meaning of colyrum?

Ans. Eye wash

Q10.what is extravasation?

Ans.

Q11.mode of action of sumtriptan?

Ans.5ht id receptor agonist

Page 111: Que & Ans_take Away

Q12.mode of action of omeprazole/

Ans.HKATPASE pump blocker

Q13.which drug cause the reabsorption of ca++

Ans.

Q14.which drug reduce the production of urate?

Ans.allopurinol

Q15.what is the structure similarity of carbacol and bethanecol?

Ans.carbacol has one more ch3 group

Q16.which condition cause the eryctile dysfunction?

a.caffine

b.smoking

Q17.what is type one error?

Ans.flase positive error

Q18.what is the structure similaritry of antihistamine?

Ans.

Q19.what is the structure difference between amphetamine and dopamine?

Ans.

Q20.which is not the side effect of morphine?

Ans.pinpoint pupil

Q21.which drug cause agranulocytosis?

Ans.rifampin

Q22.what is the metabolism of salicylic acid?

Ans.

Page 112: Que & Ans_take Away

Q23.which metabolic enzyme is most abundant in human being?

Ans.cyp3a4

Q24.which phase 1 metabolsim occour in stomach>

Ans.hydrolysis

Q25.which enzyme in mostly use for drug metabolism?

Ans.cyp3a4

Q26.mercaptepuric acid is metabolise by which metabolism?

Ans.

Q27.what is the metabolite of alcohol?

Ans.aldehyde

Q28.what is the similiraity hemoblogulin and cyp3a4 enzyme?

Ans.both have heam group

Q29.what would happen reaction between ab+ if u add a more?

Ans. See shargel

Q30.which is not the part of lymphatic system?

Ans. Thyroid gland

Q31.acth hormone is secreted from which part of brain?

Ans.anterior pituitary

Q32.where the opoid receptor r found?

Ans.

Q33.which hormone sustain the action of corpus lutum after pregnenacy?

Ans.progesterone

Page 113: Que & Ans_take Away

Q34.what r the different types of hernia?

Ans.

Q35. what r the different types of burns?

Ans.

Q36.what is the dose of alandronate?

Ans. 70mg/week

Q37.what is emphysemia?

Ans.destruction of the air way

Q38.denatrurisation of protein?

Ans.

Q39.whch drug is given in excresise induced asthma?

Ans.short acting b2 agonist

Q40.what is influenza vaccine?

Ans.live attenuated vaccine

Q41.toxic shock syndrome is caused by which organism?

Ans.s aureu

Q42.nosocomia infection is caused by which organism?

Ans.

Page 114: Que & Ans_take Away

Q43.what r the causes of osteoporosis?

Ans.

Q44.which is the side effect of raloxifine? K type

i.rash

ii.leg cramps

iii.dermatitis

ans I only

Q45.which drug is approved in Canada as monoclonial antibody?

Ans.filgristim

Q46.who quote the price of otc drugs?

Ans. Pharmacist

Q47.what is the composition of insulin?

Ans70/30

48.whiach statement is true regarding the storage of insulin?

Ans. Don,t freeze

49.doc of athlete foot?

Ans.tolnaftate

Q50.which is most stable penicillamine?

Ans.

Q51.which maintains the integrity of skin?

Ans.

Q52.moade of action of coxii inhibitor?

Ans.

Page 115: Que & Ans_take Away

Q53.what is salicylate toxicity?

Ans.respiratory alkalosisi

Q54.problems on auc?

Q55.problems on t1/2?

Q56.problem on crclerance?

Q57.problems on relative risk reduction?

Ans.see pebc exam booklet every year these questions repeate

Q58.Which drug cause the photosensitivity?

Ans.

Q59.what is lymus amibicite test?

Ans.pyrogen test

Q60.what is the moratlity measurement for smoking?

Ans see leon shargel

Q61.which drug causes venous pooling?

Ans. Nitrates

Q62.what is not the requirement of otc solution?

Ans.

Q63.structure of digoxin?

Ans. It contain sugar moiety

Q64.maximum blood supply goes to which organism?

Ans. Liver

Page 116: Que & Ans_take Away

Q65.in left ventricular failure he max blood supply goes to which organism?

Ans.lungs

Q66.which is the ophthalmic antioxidant?

Ans? See leonshargel otc cahper of eye and ear an memorise all the antioxidants use din eye preparartion.

Q67.which hormone does not secrete the secretion in stomach?

Ans.

Q68.structure similarity between clozapine and chlorpromazine?

Ans.

Q69.which is the contraindication of metformin?

Ans

Q70.what is isotonic solution?

Ans.

Q71.which is the precursor of platelets?

Ans. Megakereblast

Q72.what is the life span of platelets?

Ans.7-11 days

Q73.problem on inventory turn over?

Ans.

Q74.problems on isotonacity?

Ans.

Q75.which is not a metabolizing enzyme?

Ans.

Page 117: Que & Ans_take Away

Q76.which statement is true regarding loading dose?

Ans. To achieve steady state concentration

Q77.counceling of ant scabies drugs?

Ans.

Q78.which statement is true regarding aids?

Ans.

Q79.which statement is used to see the profitibilty of pharmacy?

Ans. Income statement

Q80.during absolute refractory condition heart muscles does not respond?

Ans. Any stimuli

Q81.which is the main organism of uti during pregnancy?

Ans. Ecoli

Q82. what is the treatment?

Ans. Flouroquinolone.

Q83.pyouria and bacteuria causes?

Ans uti

Q84.causes of bed sores?

Ans.pressuer atrophy

Q85.what is another name of herpes simplex virus?

Ans. Shingles

Q86.what do u mean by pharmacoeconomic?

Page 118: Que & Ans_take Away

Ans.

Q87.edta is used in which toxicity?

Ans.lead

Q88.which is used in propellants?

Ans.

Q89.bezylconium chloride is used as eye?

Ans.preservative

Q90.which is the shortest acting bzd?

Ans. Triazolam

Q91.bzd and barbiturates acts on which receptor?

Ans bzd receptors

Q92.what is the meaning of qlay?

Ans.quality adjusted life per year

Q93.which enzyme is responsible of rna replication?

Ans.

Q94.gas gangere is caused by whaich organism?

Ans.

Q95.which statement is true regarding moa and tyramine?

Ans. Hypertensive crisis

Q96.insulin demand increase in which condition?

Ans. Exercise

Q97.which is most potent pain mediator?

Page 119: Que & Ans_take Away

Ans.bradykinin

Q98.what are the factors which aggravates osteoporisis?

Ans.

Q99.which drug should be given in the asthmatic shock syndrome?

Ans.epeniphrine

Q100.which condition does not occour in hypovolmia?

Ans.odema

Q101.which changes the colour of urine and feces?

Ans.rifampin

Q102.which drug can be given in acute asthma?

Ans.b2 agonist

Q103.what is thixotrophy?

Ans.

Q104. whiach statement is true raegaring amphoterecin b?

Ans. It is inactive orally

Q105.what is present in inner most layer of plaque?

Ans.

(7)

Page 120: Que & Ans_take Away

1.An enzyme is a substance which acts as a catalyst.

2.A substance found commonly in fermented foods which can be toxic when MAO inhibitors are used is tyramine.

3.MAO is classified as an enzyme.

4.The anticonvulsant carbamazepine interferes with thyroid function test.

5.An excess of nitrogenous waste in the blood causes azotemia.

6.Vitamin C may give a false + glucose with Tees – Tape.

7.The heat labile factor in the vitamin B complex is thiamine.

8.Vitamin K is necessary for formation of prothrombin.

9.The average life of a red blood cell is about 4 months.

10.The precursor of vitamin A are carotenes.

11.The feeding of vitamin B12 to pernicious anemia patients overcomes the lack of extrinsic factor.

12.Vitamin E is used occasionally as an antioxidant.

13.Death due to cyanide poisoning results from cyanide inhibiting cytochrome oxidase.

14.Phenobarbital stimulates hepatic microsomal enzymes.

15.Vitamin D has a structure similar to the steroids.

16.End product of anaerobic respiration is lactic acid.

17.Mitochondria is the power house of the cell.

18.The major hallucinogenic component of marijuana is tetrahydrocanabinol(THC).

19.Coenzyme A or aryl transferase is involved in glycine conjugation.

20.GABA functions as a neurotransmitter.

21.Inositol triphosphate can act as second messenger.

22.Myxedema is a malfunction of thyroid.

23.The effect of vasopressin on the kidney is that it is antidiuretic.

24.Absence of antidiuretic hormone in the body causes diabetes insipidus.

25.The enzymes involved in ethanol metabolism are primarily alcohol dehydrogenase.

26.The end product of purine metabolism is primarily uric acid.

27.Vitamins act as coenzyme for metabolic processes.

28.Uracil is not found in DNA molecule.

29.Ketone bodies accumulate due to long or acute starvation and hypoglycemia.

30.The rich source of vitamin A is fish liver oil.

Page 121: Que & Ans_take Away

31.Vitamin D3 acts as a hormone.

32.The Michaelis – Menten equation will appear first order when the substance concentration is much smaller than Km.

33.Concentration of a drug is helpful in calculating volume of distribution.

34.During constant rate of intravenous infusion at steady state condition, rate of elimination equals constant rate of infusion.

35.Metabolism of a drug which is mainly metabolized in liver depends on intrinsic clearance and blood flow.

36.Rate of elimination is decreased by increased renal reabsorption.

37.The relative bioavailability of a drug product can be determined by comparing AUCs and total drug urinary excretion.

38.Drug products can also be evaluated by comparing curves of serum concentration vs time. The most important parameters for comparison that can be obtained from such curves are peak concentration, time of peak concentration and total AUC.

39.The area under the serum concentration – time curve represents the amount of the drug absorbed.

40.The excretion of a weakly acidic drug (eg, pKa of 3.5) will be more rapid in alkaline urine than in acidic urine because the drug will exist primarily in the ionized form, which cannot be easily reabsorbed.

41.If a fixed dose of a drug that is eliminated by first order kinetics is administered at regular intervals, the time required to achieve a steady state plasma level depends only on the half life of the drug.

42.The volume of distribution of a drug is a mathematical relationship between the total amount of drug in the body and the concentration of drug in the blood.

43.If a drug appears in the feces after oral administration, parenteral administration of the drug may determine the contribution of the biliary system to the amount of drug in the feces.

44.Low clearance drugs are affected by increase in plasma protein binding by slow process.

45.Rate of reaction for first order kinetic is dc/dt = -kc.

46.To maximize its bioavailability, pencillin G should be given on an empty stomach 1 hour before meal.

47.Biological half life does not influence the rate of absorption from solutions to any significant extent.

48.According to pharmacokinetics degradation of insulin occurs in the liver.

49.In 3 months old Jamey Scott, bilirubin is 99.9% bound to plasma proteins when the attending total plasma bilrubin concentration is 9mmol/L (0.5 mg/L). This means that 0.1% of total plasma bilirubin is not protein bound.

50.Binding to plasma protein, o/w partition coefficient and pKa of the drug are important factors in determining the concentration of drug which would be reached in the CSF treatment of meningitis.

51.Physiochemical factors affecting the rate and extent of percutaneous drug absorption include the partition coefficient of the drug molecule, the presence of surfactants in the formulation and the type of ointment base.

52.If the total body clearance of a drug is 200 mL/min in a normal healthy adult and the renal clearance is 10ml/min then one may assume that the drug is extensively metabolized.

53.A drug replaced from protein binding undergoes metabolism, shows pharmacological effect and good renal clearance.

54.Drug excreted by tubular excretion is penicillin.

55.Concentration of a drug in breast milk exceeds that in plasma, if the drug is basic.

56.With respect to bio*****alance, the parameter Cmax is affected by both rate and extent of absorption.

57.Most drugs are metabolized or eliminated from the body by first order kinetics. This implies that the amount of drug metabolized or eliminated changes with time and is dependently only on the concentration of drug in the blood.

Page 122: Que & Ans_take Away

58.The difference between peak and trough concentrations is greatest when a drug is given at dosing intervals much longer than the half life.

59.Nitrites (organic cyanides) are hydrolyzed to acids when they are heated with aqueous base or acids.

60.The reaction between phenol and aqueous solution of NaOH is considered acid – base reaction.

61.Hexaresorcinol of larger particle size is better than resorcinol.

62.In HPLC, stationary phase is non polar and mobile phase is polar.

63.Ammonium chloride is an example of acidifying agent.

64.1 – methyl cyclopentane is not an isomer of hexane.

65.F test, Chi square test and Latin square are statistical techniques for clinical trials.

66.In gravimetric analysis, the purpose of carrying out a chemical reaction to form an insoluble derivative which is then filtered off, dried and weighed is to ensure that certain potential impurities remain behind in solution.

67.In paper chromatography, the cellulose fibers have a strong affinity for the water in the solvent system and have a weak affinity for the organic solvent in the solvent system.

68.If the mean, mode and median are same the population is uniformly distributed.

69.Pyconometer is an instrument which measure weight of the same volume of different liquids.

70.Two solids melting at room temperature are eutectics.

71.From the original dye N – dealkylation metabolism occurs.

72.In a gravimetric analysis, pH is adjusted to avoid the precipitation of impurities and facilitate maximum precipitation.

73.Amorphous means no specific size and shape.

74.Dimethyl formamide is used in the assay of non – aqueous titration of sulfonamides.

75.Co – precipitate occurs by adding rapidly precipitant.

76.Potency of penicillin is expressed in international of units per mg.

77.NH4 + OH- = NH3 + H2O, is an example of neutralization.

78.CH3CH2NH is more amphiprotic.

79.Order of dissociation in water is COO->COOH->OCH3>CH3.

80.Primary amine undergoes acetylation.

81.Absorption of water is increased by adding a solvent & the process is called partition coefficient.

82.Much of formation of prodrugs is by bioactivation.

83.Free radicals are H2O2, O2 and OH.

84.2 – butene has geometric isomerism.

85.Two immiscible layers are separated by fractional distillation.

86.The molecular structure of clonidine contains a heterocyclic ring which is best described as imidazoline.

87.Folic acid has tetrahydrofolate coenzyme activity which is based on the pteridine ring.

88.H2C = CH- is called a vinyl group.

Page 123: Que & Ans_take Away

89.Both isoxazole and oxazole rings are found as substituents in some bacteriostatic sulfonamides. They are present in these structures specifically because of their electron withdrawing effects.

90.Phenazopyridine is an azo dye.

91.A carbon atom that has four different groups attached to it is an asymmetric carbon atom.

92.Structurally, caffeine belongs to a class of nitrogen bases called purines.

93.Nicotine is a nitrogen containing base with two types of heterocyclic rings, a pyridine and pyrrolidine.

94.The equation for a straight line is a = mx + b. M represents the slope of the line.

95.Structure activity relationship of antihistaminic must have terminal tertiary amine and hydrocarbon must have one or more than one aryl or hetero aryl group or its *****alent.

96.Disintegration of dx/x is lnx.

97.lne-x is –x.

98.Ninhydrin is used to identify amino acids.

99.Conformational isomer means rotation around a single bond.

100.If pKa of phenobarbitone is 7.4, 90% of the drug will be ionized at pH 8.4.

101.The reaction R – CONH – R = RCOOH + NH2R, is called deamination.

102.The temperature above which all the concentrations of a solute are soluble in water is called critical solution temperature.

103.Azeotropic mixture of two volatile liquids in which the azeotrope has a higher vapor pressure than the individual component, then azeotrope will distill faster.

104.99mTc does not eliminate from the lungs in 30 days.

105.In sulfonamide titration the role of dimethyl formamide is basic solvent.

106.Cellulose retains water in paper chromatography.

107.In TLC, organic phase is moving and silica gel is stationary.

108.In statistics, when the mean, median and mode are the same, the distribution is called normal.

109.In statistics, if 95% is standard deviation, the error is 2.5.

110.When atmospheric pressure is equal to the vapor pressure it is called boiling point.

111.A line parallel with X – axis will have zero slope.

112.The error is always smaller than the standard deviation.

113.In TLC, organic phase is moving and silica gel is stationary.

114.For a radio isotope to be useful as an scanning agent, transmission must be gamma rays.

115.HPLC resolute means scanning of one analyte from others in substance mixture.

116.In HPLC particle size affect resolute.

117.Mechanism of resolution of organic solvent in TLC depends on capillary action.

118.In paper chromatography cellulose fibers have increase affinity for water and decrease affinity for organic solvents.

119.Benzodiazepine can be metabolized by reduction.

Page 124: Que & Ans_take Away

120.Aromatic amines and hydrazides are metabolized by acetylation.

121.Capacity of a buffer to resist, change in pH upon addition of strong acid or base is due to common ion effect.

122.At excited state Boron is 1s2, 2s1, 2p1.

123.Cyclohexane is not an isomer of hexane.

124.2 – butene gives cis – trans isomerism.

125.Superimposible is not true about optical isomers.

126.Capillary action takes place in TLC.

127.In assay of amphetamine in plasma it can be extracted by ether.

128.In assay of epinephrine – HCl, tablet is crushed, dissolved in HCl, filter to extract to form base, methylene chloride is added and before extraction 0.1 NaOH is added.

129.Dipole bond does not exist in ester – ester linkage.

130.Hydrolysis of fixed oil gives glycerol + free fatty acids.

131.Electrophoresis is used to separate proteins – amino acids.

132.Pyridium is an azo dye.

133.Soap is a form of sodium salt of fatty acid.

134.Most of the local anesthetics are esters of p – aminobenzoic acid.

135.Determination of iodine value depends upon addition of iodine at the double bond of fatty acids.

136.Most of the local anesthetics are esters of p – aminobenzoic acid.

137.The rate of zero order reactions is independent of concentration.

138.Enantiomorphs are two isomers that rotate plane polarized light equal amounts in the opposite direction.

139.Iodine is used in Lugol’s solution.

140.Zinc undecylenate is used in Desenex.

141.Zirconium salt is used in Ziradryl.

142.Glyceryl guaiacolate is used in Robitussin.

143.Boric acid is used in Collyrium.

144.Grignard reagent usually contains Mg.

145.In HIV, CD4 can be detected in blood of patients, macrophages and monocytes act as reservoir for virus to protect it from body immune system and T cells are destroyed.

146.Clostridium and enterobacter are example of anaerobic organisms.

147.Limulus amebecyte is a pyrogen test.

148.Difference between gram positive and gram negative bacteria is gram +ve. stains blue and have teichoic acid in their cell wall while gram negative bacteria stains red.

149.Mycoplasma are resistant to antibiotic penicillin because it has no cell wall.

150.In UTI, organism is isolated after taking mid stream urine.

Page 125: Que & Ans_take Away

151.Nosocomial infection can be prevented by using aseptic procedures.

152.Organism of Nosocomial infection is Klebsiella pneumoniae.

153.Cell membrane is a common damage to both human and bacterial cells with the use of antibiotics.

INSHAALLAH

154.Schistosoma mansoni causes liver cirrhosis.

155.Ampicillin is acid resistant but penicillinase sensitive.

156.Cloxacillin is both acid and penicillinase resistant.

157.Staph. aureus has techoic acid in its cell wall.

158.In streptococcal infections, penicillins should be continued for 10 days.

159.Tinactin contains tolnafate.

160.A drug used as a preventative when travelling to areas in which malaria is endemic is chloroquine phosphate.

161.The causative organism of syphillis is Treponema pallidum.

162.Staph. aureus and Strept. Pneumoniae are gram positive cocci.

163.Neisseria gonorrheae is gram – negative organism.

164.The drug of choice for vaginal trichomoniasis in a nonpregnant woman is metronidazole.

165.Lyme disease is caused by Borrelia burgdorferi.

166.IgG is the most prevalent immune globulin in the body.

167.The disease vericella is commonly known as chickenpox.

168.Gas gangrene is commonly caused by clostridia.

169.Pyoderma is a pus forming skin infection.

170.Alpha hemolysis is characteristic of scarlet fever strains of streptococci.

171.Chloroquine is given weekly intervals as a prophylactic for malaria.

172.The Sabin vaccine is superior to Salk vaccine because it can be administered orally.

173.Ferments lactose and sucrose with gas production does not apply to Salmonella. typhi.

174.Koplik’s spots are associated with measles.

175.Undulant fever is caused by microorganisms of the genus brucella.

176.Tinea is a yeast.

177.In case of dental problem, cause is due to Streptococus viriden.

178.In Canada the infection of eye in neonates is due to Chlamydia trachomatous and is treated by silver nitrate eye wash.

179.Meningitis is caused by Neisseria meningitidis.

Page 126: Que & Ans_take Away

180.Toxic shock syndrome is due to Staph. aureus.

181.Gram negative bacteria retains red color and have lipopolysaccharide, peptidoglycan in their cell wall.

182.In UTI, the organism mostly involved is E. coli.

183.Diphtheria is caused by cornybacterium diphtheriae.

INSHAALLAH

184.Causative organism for endocarditis is Strept. viriden.

185.Precursor of platelet is megakaryoblast.

186.Life of platelet is 7 – 10 days.

187.Popliteal is present in knee (back space of the knee).

188.Glucagon counteract the action of insulin.

189.Retinopathy, neuropathy, and coronary artery disease are all complications of diabetes.

190.Internal carotid artery supplies blood to the brain.

191.Leukotrienes are chemical mediators and are derivative of arachidonic acid.

192.Inositol triphosphate is the second messenger in cell membrane produced by phospholipase C. The second messenger or system allows signals from cell surface receptors to be converted and amplified into a cellular response.

193.Heartburn is an esophageous symptom, occurs at night, result from the refluxof gastric *******s into the stomach. Heartburn is aggrevated by several factors, including, lying down or bending over and high fat, chocolate, spicy products, wearing tight clothes, caffeine and smoking.

194.Tetrodotoxin is a highly neurotoxic substance, its ingestion results in malaise and dizziness followed by ataxia, convulsions, respiratory paralysis and death.

195.Nicotinic cholinergic receptors are regulating ion channels.

196.Addison’s disease is a syndrome resulting from insufficient production of hormones from the cortex of the adrenal gland (glucocorticoids & mineralocorticoids).

197.An aneurysm is bulging of the wall of an artery, a vein or the heart.

198.The precursor of collagen is vitamin C.

199.Lack of ACh causes Alzheimer’s disease.

200.At rest the B. P. is lowest with supine position in arteries, venules and vena cava.

201.The common bile duct drains bile into the second part of the duodenum

202.When parietal cells are stimulated, they secrete HCl and intrinsic factor.

203.The most important complication of venous thrombosis is pulmonary embolism with resultant infarction.

204.The ability of hemoglobin to carry oxygen decreases in CO poisoning, CO2 and iron poisoning.

205.Osmotic pressure of blood is 280 mOsm/L.

Page 127: Que & Ans_take Away

INSHAALLAH

206.In cardiovascular physiology, LaPlace’s law relates the tension achieved in a blood vessel wall to the radius of vessel and the intraluminar pressure.

207.Tetrodotoxin is a selective inhibitor of the sodium ion channel.

208.Enkephalins are peptides that exert actions resembling those of opiates.

209.The pathway of blood flow from the small intestine to the liver is mesenteric vein to portal vein.

210.Schizophrenia is characterized by long - standing paranoid delusion.

211.Wernicke’s encephalopathy is caused by a thiamine deficiency.

212.Possible causes of hypercalcemia include malignancy and Paget’s disease.

213.The most likely causes of ventricular arrhythmias are potassium depletion and magnesium depletion.

214.The outer layer of the eye is composed of the cornea.

215.Patients with noninsulin dependent mellitus usually have tissue resistance and reduced number of insulin receptors.

216.A normal creatinine clearance for an adult male is 120 mL/minute.

217.Tinea versicolor is a fungal infection of the stratum corneum.

218.The layer of skin that has the greatest effect on percutaneous absorption of drugs is stratum corneum.

219.Possible cause of secondary hypertension include renal disease, oral decongestant and fenfluramine.

220.Grave’s disease may be characterized by hyperthyroidism.

221.Gamma globulin is a protein fraction of the blood.

222.Crohn’s disease is a disease of the colon.

223.Progressive fibrosis and scarring of the liver are known as cirrhosis.

224.Major constituents of bile are micelles.

225.Hodgkin’s disease is usually centered in the lymphatic system.

226.Rods are responsible for night vision.

227.Addison’s disease is caused by dysfunction of adrenal cortex.

228.Fibrinogen is synthesized mainly in the liver.

229.The effect of the antidiuretic hormone is to increase the rate of resorption of water by the kidney.

230.An endocrine gland that plays an important role in calcium metabolism is parathyroid.

231.A major factor in control of sodium excretion by the kidney is aldosterone.

232.Melanin and smooth muscle are not found in dermis.

233.Edema is the excessive accumulation of the watery fluid in the tissues.

234.Coronary artery receives blood from aorta.

Page 128: Que & Ans_take Away

235.A substance that naturally occurs in the body is called endogenous.

236.Colon has the most basic pH.

237.Intrinsic factor is present in parietal cells of blood.

238.Ascites means peritoneal retention of fluid.

239.Schiatic nerve supplies muscles to the calf and back of thigh, skin of lower calf and the upper surface of foot.

240.Epistaxis is bleeding of nose.

241.Glutathione protects cell integrity.

242.Causes of tachycardia are increased temperature, stimulation of heart autonomous nerves and toxic condition.

243.In the post menopausal therapy, conjugated esterogen carry the risk of endometrial cancer.

244.Arrythmia with respiration and inspiration is called paroxysmal arrythmia.

245.When left ventricle valve has been blocked, lungs get more blood.

246.Hernia is a protrusion of the peritonium.

247.Compared to interstitial fluid, plasma has higher concentration of protein.

248.Onset of CHF (left) is manifested by accumulation of blood in lungs.

249.CHF (right) is manifested by blood accumulation in vena cava.

250.In a cell network runs throughout the cytoplasm responsible for enzymatic reaction is endoplasmic reticulum.

251.Parietal cell intrinsic factor is vitamin B12.

252.In organ transplant, the rejection is due to infiltration of T – cells.

253.Normal blood flowing through the body contains prothrombin.

254.Structure of intestine consists of jegenum, ileum and duodenum.

255.Liver receives maximum cardiac output (27%) at rest.

256.Adrenal gland does not secrete in GIT.

257.The brain receives most of the blood supply from internal carotid artery.

258.50% of the normal blood plasma protein is albumin.

259.Skeletal muscle is not included in dermis.

260.During inspiration low blood pressure is in vena cava.

261.Signs and symptoms of tardive dyskinesia are oral buccal signs.

262.Epilepsy is the result of GABA deficiency.

Page 129: Que & Ans_take Away

INSHAALLAH

263.Thyroid is not a part of lymphatic system.

264.Gravida is the number of times someone has become pregnant.

265.Methyl alcohol causes toxicity to retina.

266.Greater absorption in small intestine than in stomach is due to greater surface area.

267.Urine, skin and sweat glands help in regulating the body temperature.

268.Hemorrhage, ascites and jaundice are complications of cirrhosis.

269.Nerves in the human body that transmit their impulses by releasing ACh are known as cholinergic nerves.

270.Insulin has a pharmacodynamic effect at cell surface due to interaction with receptors.

271.A drug used to treat delirium tremens is chlordiazepoxide.

272.Pyrivenum is an antehelmintic.

273.Allopurinol must be used with care with mercaptopurine.

274.Prostaglandins and Tamoxifen are used to terminate pregnancy.

275.Antineoplastic drugs are given by i.m. and i.v. routes in chemotherapy.

276.Piroxicam decreases prostaglandin synthesis.

277.Addiction does not occur in case of antitussive doses of codeine.

278.Cortisone is a modified antirheumatic drug.

279.Giving a vasoconstrictor one hour before introducing anesthesia means; decrease blood supply enhance long action of anesthesia, decrease side effects and give smoother and good induction of anesthesia.

280.Losartan, is an Angiotensin II receptor inhibitor, inhibits, both the vasoconstrictor and aldosterone – secreting effects. It is used in mold to mild hypertension and contraindicated in pregnancy.

281.Oxazepam is an intermediate acting benzodiazepine.

282.Nalbuphine produces opioid withdrawal symptoms if given to morphine dependent patients.

283.Flumazenil is a benzodiazepine antagonist acts on GABAA receptor.

284.Benzodiazepine acts on GABAA receptor.

285.Sumatriptan is a serotonin agonist act by 5 – HT cell membrane receptor type, having side effects like dizziness, muscle weakness or neck membrane.

286.Celebrex (Celecoxib) is a NSAID drug acts by inhibiting cyclooxygenase II.

287.Extrapyramidal symptoms are side effects of neuroleptics

288.Gastrointestinal disturbances are the side effects of indomethacin.

289.Acetaminophen is given to the patient for antipyretic and analgesic action.

290.Nifedipine, diltiazem, and verapamil are calcium channel blockers that cause peripheral edema.

291.Thiopental sodium crosses the BBB rapidly.

Page 130: Que & Ans_take Away

292.Clonidine causes rebound hypertension.

293.Toxicity of acetaminophen is hepatic cirrhosis.

294.Magnesium hydroxide is a non systemic antacid with cathartic side effect

295.Pancuronium is a competitive inhibitor of Ach.

296.Contraindication of oral contraceptive is pregnancy.

297.Long-term use of corticosteroid causes Cushing’s syndrome and osteoporosis.

298.Cortisone does not cause hypoglycemia.

299.Omeprazole is a proton pump inhibitor.

300.In maniac depression, lithium carbonate is given.

301.Side effects of thiazide diuretics are hypokalemia, hyperurecemia, hyperglycemia, hyperlipidemia, hyponatremia and hypercalcemia.

302.Furosemide is classified as loop diuretic.

303.An oral antifungal agent is griseofulvin.

304.Weak acid (acetic acid) can be given in base poisoning.

305.Malignant hyperthermia can be caused by general anesthetics.

306.Trimethoprim acts synergistically with sulfonamides.

307.Morphine tolerant patient needs analgesia, pharmacist give him methadone.

308.Side effects of local anesthetics due to amide cause convulsion, hyperthermia and CNS effects.

309.Feldene (Piroxicam) can be given once a day.

310.Ascorbic acid causes a positive Coomb’s test.

311.Action of inhalation anesthetic is based upon concentration in brain.

312.Metabolism of local anesthetic takes place in liver.

313.In bleach, gasoline and organic acids poisoning there should be no emesis.

314.In absence of embolism, the drug used in the thrombolysis is streptokinase.

315.Neuroleptics act by inhibiting dopamine receptor. Therefore not use in Parkinsonism.

316.Acetazolamide causes hypochloremia.

317.Chemical classification of nifidipine is dihydropyridine.

318.Acetaminophen is least used in acute gout.

319.Salicylaism is also called as Reye’s Syndrome.

320.Viagra (Sildenafil citrate) is contraindicated in patients who are on any form of nitrate drug therapy.

321.Vincristine acts by inhibiting spindle formation in cell division.

Page 131: Que & Ans_take Away

322.Omeprazole acts by inhibiting H+- K+ ATPase.

323.Short acting benzodiazipine is triazolam.

324.Atropine is the primary example of an important class of muscarinic antagonists which are esters of tropic acid and which contain pyrrolidine and piperidine.

325.In circular muscles of the iris, the pharmacological action of nicotine is not described as nicotine.

326.Erythromycin estolate, is a macrolide antibiotic, primarily effective against bacteria & adequately absorbed from the gastrointestinal tract.

327.The major metabolites of diazepam have therapeutically significant pharmacological activity.

328.Mechanism of drug interactions with acetylsalicylic acid include displacement of other drugs bound to serum albumin.

329.Phenylephrine does not increase the heart rate directly or by reflex when injected intravenously into a young healthy adult.

330.Acetazolamide, a diuretic acts by blocking the Na+ - H+ exchange system of the renal tubule.

331.Diarrhea is not a sign of atropine poisoning.

332.Pernicious anemia is treated by parenteral administration of vitamin B12.

333.Sulfinpyrazone is used as a uricosuric agent.

334.The shock and airway edema of anaphylaxis are best treated with adrenaline.

335.Flumazenil is a specific antagonist for diazepam.

336.Cetrizine is histamine 1 antagonist.

337.Sumatriptan’s efficacy in migraine therapy is attributed to selective 5 – HT1 agonist.

338.Anticholesterol drug for pregnant women is cholestyramine.

339.Pepto-Bismol causes black stools.

340.Best antidote for mercury poisoning is sodium formaldehyde sulfoxylate and egg albumin is given in emergency.

341.Phentolamine (Regitine) is used to prevent local necrosis from norepinephrine infusions.

342.Thiazide diuretics may produce an increase in blood levels of uric acid and glucose.

343.Fluorouracil (Adrucil) is classified as an antimetabolite.

344.A drug that is effective in the treatment of alcohol withdrawal syndromes and in the prevention of delirium tremens is chlordiazepoxide (Librium).

345.Cromolyn sodium (Intal, Nasalcrom, Opticrom) is a drug that is a stabilizer of sensitized mast cells.

346.A pharmacist receives a prescription order for indomethacin (Indocin) capsules. He should consult with the physician if the medication record indicates that the patient has a peptic ulcer.

347.Phenobarbital is a microsomal enzyme inducer.

348.The thiazide diuretics decrease the excretion of uric acid.

349.Carbon monoxide exerts its effects primarily by decreasing the oxygen carrying capacity of the blood.

350.The anti – inflammatory effect of aspirin is due to inhibition of synthesis of prostaglandins.

351.Tamoxifen (Nolvadex) is an agent that can best be described as an antiestrogenic.

352.Pilocarpine is employed in the treatment of glaucoma. It is believed to act as a miotic.

Page 132: Que & Ans_take Away

353.Stimulation of gastric secretion is not an effect of atropine on the human body.

354.Dicloxacillin is the most useful in the treatment of an infection caused by beta – lactamase – producing staphylococci.

355.Methysergide is a serotonin antagonist.

356.Carbamazipine (Tegretol) is the drug of choice for trigeminal neuralgia.

357.After oral administration, the greatest amount of iron absorption occurs in the duodenum.

358.Iron is required by the body to maintain normal oxygen transport.

359.The primary site of action of triamterene (Dyrenium) and spirolactone (Aldactone) is the distal tubule.

360.Acid rebound most likely to occur with the use of large doses of calcium carbonate.

361.Ondanesetron (Zofran) is indicated for use as an antiemetic agent.

362.Methadone is not an addicting drug.

363.An example of pure narcotic antagonist is naloxone HCl (Narcan).

364.A uricosuric drug is one that promotes excretion of uric acid in the urine.

365.Aminophylline is used most frequently in asthma.

366.Clofibrate is used to lower blood lipid levels.

367.Rifampin is a first line drug to treat tuberculosis.

368.Antidiuretic effect, constriction of capillaries, coronary vasoconstriction and increased motility of bowel are all characteristics of vasopressin.

369.A specific narcotic antagonist is nalorphine.

370.Ritodrine (Yutapar) is used to control preterm labor.

371.A very common side effect of morphine is constipation.

372.The greatest threat from morphine poisoning is respiratory depression.

373.Parkinsonism is probably due to too little dopamine in the brain.

374.Heparin is the fastest acting anticoagulant.

375.Sulfones such as dapsone are employed commonly in the treatment of leprosy.

376.Doxycycline (Vibramycin) has the longest duration of action.

377.Penicillins are believed to exert their antibacterial effect by inhibition of bacterial cell wall synthesis.

378.Oxytocin is released from the posterior pituitary gland.

379.During ovulation, peak plasma concentrations of the FSH and LH will reached.

380.Stimulation of gastric secretion is a pharmacologic action of histamine.

Page 133: Que & Ans_take Away

381.Gold (metal) has been found useful in the treatment of rheumatoid arthritis.

382.As an antiarrhytmic drug, procainamide is most similar in action to lidocaine.

383.Acetylcysteine is an agent used clinically as a mucolytic.

384.Epinephrine may be added to local injections of lidocaine HCl in order to prolong its effect.

385.Heparin therapy is best controlled with clotting times.

386.Pilocarpine produces constriction of pupil of the eye.

387.The mechanism explaining the clinical picture observed in carbon monoxide poisoning is a chemical union of the carbon monoxide with the hemoglobin of the red blood cells.

388.EDTA is an antidote for lead.

389.Epinephrine HCl is used intravenously in a concentration 1:1000.

390.Epinephrine is often included in the administration of local anesthetics because it delays diffusion of the anesthetic from the site of injection.

391.The toxicity of methanol is due to its conversion in the body to formic acid and formaldehyde.

392.Alcohol in the body is oxidized to CO2 and water.

393.Polycarbophil can be used to treat both constipation and diarrhea.

394.Methanol may cause blindness.

395.The emetic action of morphine is due to stimulation of emetic chemoreceptor trigger zone.

396.Colchicine is used mainly to treat gout.

397.Terbutaline has a preference for stimulation for Beta 2.

398.Streptomycin can cause eighth cranial nerve damage.

399.Hyperstat IV (diazoxide) is used mainly in treating emergency reduction of blood pressure.

400.Emulsions made with tween are usually o/w.

401.A liquid whose viscosity is increased when stress is applied is classed as dilatant.

402.Fatty tissues and protein tissues frequently store drugs.

403.Large apparent volume of distribution is the first evidence that a drug is being stored in tissues.

404.Oxytocin is used to induce labor.

405.Penicillin G undergoes marked hydrolysis in the GIT.

406.Surface active agents tend to enhance absorption due to effects on biological membrane, effects on dissolution rate of drugs and reduction of interfacial tension.

407.Plasma protein binding slows the process of biotransformation.

408.If a CNS drug is extensively ionized at the pH of blood it will penetrate the BBB slowly.

409.The solubility of a chemical in a given solvent is influenced by many factors that are dielectric constant, pH of solution, pKa of the chemical and solubility parameters.

410.Edetate is a chelating agent.

411.The process of grinding a substance to a very fine powder is termed as trituration.

Page 134: Que & Ans_take Away

412.The type of flow in which the viscosity of a liquid increases with agitation is dilatant.

413.Sodium chloride 0.9% injection is the most appropriate for an IV admixture of ampicilin (500mg/50mL).

414.Sodium lauryl is a wetting agent.

415.The major mechanism of degradation of drugs in the GIT is hydrolysis.

416.Oxidation is not a major pathway or type of biotransformation.

417.The form of water most commonly used as a solvent during the manufacture of parenterals is water for injection USP.

418.A suspension is not a suitable dosage form for intravenous.

419.Ascorbic acid possesses antioxidant properties.

420.Colostomy pouches are classified by an open Vs closed designs and size of stoma.

421.Glycerin is the most hygroscopic liquid.

422.The HLB system is most applicable for the classification of nonionic surfactant.

423.An example of nonionic surfactant would be sorbitan monopalmitate.

424.The HLB system is used to classify surfactants.

425.Passive diffusion is usually slow.

426.Glucagon may be used as a diagnostic agent in radiographic examination of the GIT.

427.The most common disintegrator in compressed tablets is starch.

428.Freons are fluorinated hydrocarbons.

429.Methotrexate should never be used concurrently with aspirin.

430.Tablet hardness range is normally 3.5 –7 kg.

431.An antidote for heparin overdosage is protamine sulfate.

432.A substance that is often used to subcoat tablets is shellac.

433.Cheese is contraindicated with MAO inhibitors.

434.Riopan product is a chemical combination of aluminum and magnesium hydroxide.

435.Poorly manufactured tablets may result in splitting off of the surface of the tablet. This phenomenon is known as picking.

436.Starch is not used primarily as a diluent in tablet formulations.

437.An ingredient that is added to a tablet formula to improve flow properties into a die for compression is known as glidant.

438.Penicillamine is often classified as a chelating agent.

439.Prostaglandin E2 is used in the form of vaginal suppositories.

440.Splitting off of the face of a tablet is called capping.

441.Constipation is a side effect of cimitidine.

Page 135: Que & Ans_take Away

442.Simethicone relieves flatulence primarily because of defoaming action.

443.The ideal antiseptic concentration of ethyl alcohol is 70 %.

444.An excellent choice of diluent for a compressed vaginal tablet formulation would be lactose.

445.Colligative properties are useful in determining tonicity.

446.Elevation of boiling point, osmotic pressure, lower of freezing point and lowering of vapor pressure are all colligative properties of solution.

447.An osmotic solution has the same osmotic pressure as blood.

448.A second method for adjusting solution to isotonicity is based upon freezing point depression.

INSHAALLAH

449.The function of papain in a soft contact lens product is to remove proteinaceous residues.

450.99mTc is considered to be an almost ideal isotope for medical applications and is commercially available as a radioisotope generator.

451.The decay of radioactive atoms occurs as a first order reaction.

452.Gamma radiations have the greatest penetrating power.

453.Surfactants are characterized by the presence of water solubilizing and fat solubilizing groups in the same molecule

454.Another name for methadone is metaphenone.

455.Aluminum acetate is used as astringent in hemorrhoids.

456.Aggregation of the suspending agent is called micelles.

457.Sodium bisulfite is an antioxidant.

458.In a suspension settling of particles can be overcome by adding non-ionic SAA, increasing viscosity by adding shear-thickening agent and reducing the particle size.

459.Suspension is a two-phase system in which the internal or disperse phase is solid and the external or continuous phase is liquid.

460.Bentonite magma is a suspending agent.

461.When less than the average pharmacist cares it is called negligence.

462.A person is using long crutches, if the radial nerve is damaged wrist drop will be effected.

463.A number of studies have been done on patient counseling methods. Both verbal and written directions have been suggested.

464.Levigating agent should form a smooth paste, should be compatible with ingredients and should be washed off easily.

465.Triethanolamine stearate is an emulsifying agent.

466.If we increase the binder in tablet manufacturing then hard granules will be formed and tablet will not be easily disintegrated.

467.Usually metabolites are having low o/w partition coefficient.

468.Mottling is uneven distribution of color.

469.Role of mineral oil in sulfur ointment is levigating agent.

470.Most frequently used fillers in tablets are dextrose, lactose, sucrose and mannitol.

Page 136: Que & Ans_take Away

471.Most frequently used binders are cellulose, starch, PVP, glucose.

472.n gel preparation bentonite is not used to increase the viscosity.

473.Aqueous antioxidant includes sodium bisulfite, ascorbic acid and EDTA.

474.Non-aqueous antioxidant includes vitamin E.

475.Poly (dl – acetic acid) does not undergo degradation.

476.Acidic drugs in plasma bind with albumin and basic drugs bind with alpha acid glycoprotein.

477.Ampicillin dissolved in 5% dextrose can be kept for 4 hours before use, 4 hours in dextrose and 8 hours in saline solution.

478.Usually metabolites are having low o/w partition coefficient.

479.Glycerin in pharmaceutical preparations is used as humectant.

480.In the solution of diazepam, ethyl alcohol is used and propylene glycol is added to prepare i.v., injection. In this solution propylene glycol acts as cosolvent and inhibitor of fermentation.

481.Intra – arterial injection is given in artery.

482.Solid to gas is called sublimation without changing into liquid state.

483.Better bioavailiability of injection can be achieved by rubbing.

484.Burrow’s solution is astringent.

485.Burrow’s solution is aluminum acetate.

486.Increase in viscosity by increasing in rate of shear is called dilatancy.

487.Ice application decreases the absorption at injection site.

488.Sterilization of certain hormone is done by radiation.

489.Thixotrophy is the term used for liquids, which flow freely if recently stirred, but gel when left undistributed.

490.Particle size of aerosol suspension is usually 0.10 – 1 millmicron.

491.The most efficient /powerful anti oxidant is d - tocopherol.

492.Benzyl alcohol 5% is used as solubilizer in oily preparation, local anesthetics, ointments and ophthalmic solutions.

493.p.c. means after meals.

494.Pharmacoeconomics has been defined as the description and the analysis of the cost of the drug therapy to healthcare systems and society.

495.Pharmaceutical care has been defined as the responsible provision of drug therapy for the purposes of achieving definite outcome.

496.Income statement is used to determine the total value of prescription drug sales for a pharmacy during the course of a year.

497.A paste containing karaya gum powder is utilized in ostomy patients to protect skin from stomal effluent.

498.Efflorescence means loss of moisture.

Page 137: Que & Ans_take Away

499.Compared to the sublingual tablet, nitroglycerin ointment provides a more prolonged effect.

INSHAALLAH

500.The dissolution rate of nitrofurantoin tablets would be decreased by increasing the force of compression during tableting.

501.Liquid paraffin is emollient laxative.

502.1 T qid pc & hs means one tablet four times a day (after meals & at bed time).

503.A person comes to the pharmacist complaining hard stool after every 2 or 3 days. Pharmacist will advise him a stool softener (Docusate sodium).

504.In parenteral preparation the major contamination is in equipment.

505.Surface active agent for contact lense improves wettability.

506.Suppository base, which has polymorphic properties, is theobroma oil (cocoa butter).

507.Triethanolamine stearate is an emulsifying agent.

508.Methyl paraben and propyl paraben are used together because both act synergistically.

509.White ointment, cetosteryl alcohol, sodium lauryl sulfate, water and mineral oil are examples of w/o emulsion.

510.Sterilization of articles with ethylene oxide requires aeration with air.

511.Dispersion of oil globules in aqueous vehicle is called emulsion.

512.When zinc oxide suppository of 200 mg is prepared where correctional factor is 4, it means 200 mg of zinc oxide will displace 50 mg of theobroma oil.

513.Sterilization of articles with ethylene oxide requires sterility, pyrogen free and free of particles.

514.Factor responsible for transfer of drug across the placental barrier is molecular size of the drug not the lipid solubility.

515.The pore size of bacterial filter is 0.22nm.

516.IM injection is preferred over IV injection to avoid capillary damage.

517.Transdermal patch releases drug by diffusion.

518.Insulin preparation can be kept at room temperature for 30 days.

519.Viscosity can be increased by adding thixotropic gel.

520.Acceptable turnover rate of prescription is 5%.

521.Antioxidants, which mechanistically act as reducing agents include ascorbic acid and sodium bisulfite.***

522.Biceps are present in arm muscle.

523.Oral metal chelator used in rheumatoid arthritis is auranofin.

524.Dissolution is increased by the temperature.

525.When alcohol and water acts together, alcohol acts as cosolvent.

526.Mercaptopurine is metabolized by glucoronic acid oxidation.

527.Complication of liver cirhhosis is ascites, GI bleeding and encephalopathy.

Page 138: Que & Ans_take Away

528.Mechanism of action of sulfinpyrazone is inhibition of platelet granule synthesis and prostaglandin synthesis which prevents platelet aggregation.

529.Alpha serum fetoprotein is increased in pancreatitis.

530.Congestive heart failure is not a complication of ischemic heart disease.

531.P.O.S means point of sale in pharmacy.

532.Difference between interstitial fluid and plasma is protein quantity.

533.Congestive heart failure represents a hypervolemic state.

534.If electron withdrawing group is in the vicinity of a weakly acidic drug then pKa will decrease.

535.Diaper rash in small babies can be treated by zinc oxide cream(zincofax).

536.Pannus is superficial vascularization of cornea with infiltration of granular tissue.

537.A drug accumulates in a dose of 200 mg with a half life of 2 hours given every 8 hours.

538.Enols are tautomeric forms.

539.In hepatic insufficiency bilirubin increases and albumin decreases.

540.Mevalonyl coenzyme A is accumulated in patient with megaloblastic anemia in folic acid deficiency and cyanocobalamine deficiency.

541.Low molecular weight substance combines with high weight substance by heptane protein conjugation process.

542.Long use of antiepileptic drugs and folic acid deficiency cause megaloblastic anemia.

543.Allupurinol increases the level of mercaptopurine by inhibiting its metabolism (allupurinol – inhibition of xanthine oxidase increases the serum concentration of mercaptopurine).

544.Pigmentation of skin does not affect absorption of drug through skin.

545.Cocaine is used as a local anesthetic and vasoconstrictor during ENT procedures.

546.Dopamine acts on a1, b1,b2, D1 and D2 receptors.

547.Nephrotic syndrome, glomerulonephritis and UTI represent bacteriuria and pyouria.

INSHAALLAH

548.If a female has not hemophilia and his husband is suffering from hemophilia then the male child will not be hemophilic.

549.Hernia is at the junction of gastroesophageal and in the strangulated hernia there is no blodd supply.

550.Heme group and oxidation is a similarity between hemoglobin and cytochrome molecule.

551.Permanent alveolar destruction occurs in emphysema.

552.The excretion of a weak acid is increased by NaHCO3.

553.Causes of hypercalcemia are hyperthyroidism, Paget’s diseae, thiazide diuretics, vitamin D intoxication and excessive calcium intake.

554.Zafirlukast is a leukotriene antagonist and used for asthma.

555.Hashimoto’s thyroiditis (autoimmune disease) is hypothyroidism.

556.W/O emulsion have 1 – 9 HLB.

Page 139: Que & Ans_take Away

557.Salicylaism is a side effect at the dose of inflammation.

558. Calcium reabsorption in distil convulated tubule is due to parathyroid hormone.

559.If a drug is 90% protein bound, an increase in the dose will effect in accumulation in the body.

560. Buffers are usually prepared by weak acid and salt of a weak acid or weak base and salt of the weak base.

561.Glutathione is a cell stabilizer enzyme.

562.Loop diuretics act on the ascending loop of Henle, inhibiting the transport of NaCl out of the tubules into the interstitial tissue.

563.Mannitol is an osmotic diuretic.

564.A patient comes to the pharmacy taking metformin 500 mg and ask pharmacist about the dose reduction. Pharmacist will advise patient to keep taking this medication and refer patient to physician before making any decision.

565.Horizontal HEPA filters are used in laminar flow for the preparation of cancerous drugs.

566.Amitryptilline and imipramine are also used in nocturnal child enuresis.

567.Prolong treatment adverse effect like pulmonary fibrosis is because of Idoxuridine.

568.Vitamin E is used as solvent in oily injection.

569.Alprastidil (PGE2) is used for erectile dysfunction.

570.Aldosterone secretion is controlled by ACTH.

571.Peripheral edema is a result of right ventricular failure.

572.Pannus is a particular indication of arthritis.

573.Mechanism of action of simvastatin is inhibition of HMG CoA reductase inhibitor.

INSHAALLAH

574.Cholestyramine is a bile acid resin positively charged ammonium polymers.

575.Cholestyramine acts by binding negatively charged bile acids in the small intestine forming insoluble complexes that then excreted in feces.

576.Bleomycin causes pulmonary fibrosis.

577.Etidronate (Didronel) is a biphosphonate, used for malignancy associated with hypercalcemia, Paget’s disease and osteoporosis.

578.Pharmacist will refer patient in blurred vision.

579.Mutagensis is change in DNA, find rapidly in dividing cells and a change in genotype.

580.Advantages of sustained release tablets are compliance to patients and decreased side effects.

581.Morphine is a drug of choice in myocardial infarction.

582.Epinephrine and norepinephrine cause hyperglycemia by increased glycogenolysis, increased glucagons secretion and decreased insulin secretion.

583.Adrenal gland, parotid gland, bile duct and ******* of the stomach all fall in the duodenum.

584.Maxillary bone, ethmoid bone, frontal bone and occipital bone all are skull bone.

585.Pelvis, ischium and pubis are all pelvic bone.

Page 140: Que & Ans_take Away

586.Long use of hydrocortisone cause decrease resistance to disease, glucose intolerance and hypokelemia.

587.Acarbose (Precose) is an alpha glucosidiase inhibitor, inhibits post prandial rise of blood glucose and decrease absorption of starch in the intestine.

588.Retinoic acid (Vitamin A) is used in acne.

589.Oral contraceptives cannot be given in liver dysfunction, seizure epilepsy, deep vein thrombosis, breast cancer and pregnancy.

590.The name of blind spot is optic disk.

591.Pancreatic cells are a, b and d cells.

592.Nitroglycerin and isorbide cause venous pooling.

593.Calamine lotion (8%) is used to treat poison ivy.

594.A decrese in ADH means increase thirst and polyurea.

595.Bradykinin, histamin, leukotriene, PGs and interleukin – 1 are algesic.

596.Antidiuretic hormone is secreted from posterior pituitary.

597.Methionine is an essential amino acid.

INSHAALLAH

598.Epidermal cells when exposed to UV radiation convert a steroid related to cholesterol into vitamin D3. This product is absorbed and after processing released from liver.

599.Deficiency of vitamin D causes rickets and and its effects increase in presence of vitamin C.

600.Diazepam has active metabolite (half – life 8 hours), its long use then stopping cause seizures and it changes the normal sleep pattern.

601.Preanesthetic medication means smooth induction of anesthesia, relieve anxiety and prevention of post – operative complications.

602.Oral iron preparations are normally preferred.

603.By – pass is not a condition of heart inflammation.

604.Extravasation means escape or discharge (blood) from a vessel into the tissue.

605.In digoxin, the death is due to left ventricular tachycardia and ventricular fibrillation.

606.Sinusitis is caused by Hemoph. Influenza and Strept. Pneumonia.

607.In chemotherapy of cancer, the routes of administration are I.M., I.V. and oral.

608.Toxicity of paracetamol is due to active metabolite (N – acetyl – p – benzoquinone).

609.In paracetamol (acetaminophen) toxicity, after giving N – acetylcysteine, the mortality is safe and can be waited for 10 – 12 hours.

610.Calamine contains combination of magnesium or zinc with ferric oxide.

611.The best method to instill eye drops is to pour drops in lower sac.

612.Creatinine in plasma is detected in renal functional test.

Page 141: Que & Ans_take Away

613.Glomerulous clearance is detected by creatinine.

614.Passage of ointment through skin depends on partition coefficient.

615.A patient is having type I diabetes and severely dehydrated. He is brought to emergency with a very high level of sugar. The treatment would be crystalline zinc insulin suspension.

616.Benzalkonium acts as preservative and bactericidal.

617.Mode of action of heparin is potentiation the action of antithrombin III which activates thrombin and other factors especially Xa involved in clotting factors.

618.Mechanism of action of warfarin is competitively antagonism of vitamin K and it takes 48 – 72 hours.

619.Vitamin K is an essential cofactor for synthesis of clotting factors II, VII, IX, X and protein C and S.

INSHAALLAH

620.In gout patient pain is treated on the priority basis.

621.Urinary tract infection is more common in pregnant women.

622.Very soluble means less than one part.

623.Polar solvents those made up of strong dipolar molecules have hydrogen bonding.

624.If a drug has a high o/w partition coefficient or a high membrane /buffer partition coefficient then it will be absorbed more rapidly across cell membranes and tissues barriers.

625.The most commonly used solvents in HPLC are hexane, isooctane, acetonitrile, methanol and water.

626.Common preservatives are chlorobutanol, potassium sorbate sorbic acid, propyl gallate and benzalkonium chloride.

627.Leukotrienes are derivatives of arachidonic acid and classed are B4, C4 and D4.

628.Emphysema means abnormal dilation of the air spaces lying beyond the terminal bronchioles accompanied by destruction of their walls.

629.Neoplastic cells can defend themselves in several ways including increased DNA repair, changes in target enzymes, drug inactivation, decreased drug accumulation and alternative metabolic pathway

(8)

The physician prescribed prednisolone 40 mg stat & taper 5 mg daily .how long the therapy will last(8 days)

Which medicine should be taken in empty stomach (sucralfate)

POS means (point of sale)

Transition metal is (Cu /Co)

Which artery supplies blood to brain (internal carotid & Vertebral)

Which drug doesn’t causes opoid withdrawal syndrome (Nalbuphene)

Page 142: Que & Ans_take Away

DIN(drug identification number)

questions related to dispencing of schedule drugs

paralex of liquid may be minimized by seeing it straight to the level of eye

UTI of women(what should pharmacist do in councilling)

numerical on inventory turn over

AT II receptor anatomist is LOSATAN

which organism causes UTI(gram- bacilli)

endocarditis caused by stphyllococus aureaus

1,3-dihyrdo pyridine is nifidipine

bed sores are caused by pressure atrophy

structure of bethancol & carbacol(difference)

structure difference b/w amphetamine and dopamine

short acting anesthetic is thiopental

sumatriptan is indole derivative

sumatriptan is 5HT1-receptor antagonist

antidote for Alprazolam is Flumazanil

antidote for cabon monoxide is Oxygen

ninhydrin is used for detection of amino acids

which drug inhibit lipoxygenase pathway(Zafirlukast)

Nosocomial infection is caused by staphylocus aureas

clymada infection in Canada (in childrens eye)

Mg stereate is used as a glidient / lubricant

question son marketing of drugs

which drug is used for prophylaxis of TB(isoniazid)

Ca++ is absorbed by the distal tubules

in metabolic acidosis the level of bicarbonate is increase

hasimoto disease is hypothyroidism

I131 is used for screening of Hypothyroidism

Parkinson disese caused by decrease in the level of dopamine

trituration is particle size reduction

pulverization with intervention is used to reduce the particle size of camphor

non allergic asthma is caused by exercise

angina s aggravated by coitus

Page 143: Que & Ans_take Away

hypertensive crises caused by MAO inhibitor

amphotericin B is hydrophilic

interaction of sildefanil and Tylenol

which drug is not aminoglycoside( Vancomycin)

diabetis test is glycosylated Hb

cytochrome is not an enzyme

which cytochrome is most abandon in the body (CYP3A4)

buccal route have not the large surface area

nasal route is high capillary permeability and blood supply

T-cells rejection is treated with Prograf(Tacrolimus)

non allergic asthma is caused by excercize

benign prostrate hyperplasia / Prostrate cancer timsulosin is used

ratio of aerobic & Anaerobic in colon (90/10,60/40,30/70)

benzodiazepines act on GABA-a receptor

meprosin low lipid

ibuprofen can be used in children

flumazanil is antidote for benzodiazepine

AB-blood group can accept blood from every body

glycosylated hemoglobin used to measure blood glucose leval

teratogenic effect occurs between 3-9 weeks

nasal pathway increases blood supply

if kidney function is greater than 20% don’t change the dose

cream for nappy rash is zinc oxide

megaloblastic anemia is caused by deficiency of vitamin B12 and folic acid

mineral oil is used as a laxative and it decrease the absorption on oil soluble vitamins

insulin can be kept outside refrigerator for 30 days

pantoprazole is not an H2-receptor antagonist(it is proton pump inhibitor)

Hepatitis –A is not transfer from person to person

drug of abuse are(caffeine,Tylenol and codeine,and biscodyl)

methotrexate dose is 25 mg ,3tab weekly

prograf (tacrolimus) kills T-cells

sumatriptan is 5HT1a receptor antagonist

in gas chromatography the carrier is gas

Page 144: Que & Ans_take Away

reasons for hyper calcemia

ligase bind DNA-receptor together

ratio of albumin and protein in the body

methyl alcohol causes blindness

asthamatic child has been given prednisolone

ACTH is released bythe pituitary

pilocarpine causes miosis

in job deligation boss tells employees what to do

medicine which are female products (for councilling the pharmacist should take the pt along with partner to private place)

steroid side effects

streatre is lubriacant

lithium is used in manica depression

antidote of warfarrin is vitamin –K

salting out –rapid stirring and cooling

heat sterilization relates questions

vitamin –D is also act as a hormone

cimitidine is not used to treat allergy ( as it is H2-receptor antagonist)

celebrex (celecoxib) is COX-II inhibitor

mesoprostol is prostaglandin E1 antagonist

soft capsules has sorbitol in them

what is burros solution

herpes simplx is chicken pox

mechanism of action of benzodiazepines

trituration decresses the particle size of substances

what are lipid levels in the healthy individuals

aneurism is the narrowing of the blood vessels

efflorescent is loss of moisture

herpes transfer by lesions

type I error is false positive

blood pressure in borderline hypertension is 160/100

free radicals are H3O,O3

question related to net income

sciatic nerve supplies to all foot region

Page 145: Que & Ans_take Away

colligative properties depend on number if particles

what is eutectic mixture

questions related to pharmacy turn over

cholystasis is liver infection

during ovulation the level of FSH and LH are increase

cromylin is used in allergic treatment

loratidine is a non sedative anti histamine

ADH act on distal convulated tubules

angina is not caused by warm weather

aspirin is hydrolysed by hydrolysis

penicillamine is a chelating drug

COPD is measured by PEFR (Peak Expiratory Flow Rate)

treatment of CO poisioning is Oxygen

what is embolism

osmotic pressure of blood is 280 mm Hg

enalapril is ACE-inhibitor

rate limiting step in transdermal delivery system is diffusion

methotrexate should not be taken with foilc acid

in elderly every thing gose down except serum cretinine

serum cretinine is measured by glomerular filtration rate

similarity between hemo globin and cytochrome is heme

vitamin that is heat labile is thiamine

iron mainily absorbed in duodenum

bone that is not a maxillary bone is hynoid

thyroid is not a part of lymphatic system

staph.aureus causes anaphylactic shock

UTI us caused by gram negative bacilli

main organisms in gut is anaerobic bacteria

stratum corneum is the rate limiting step in skin drug absorption

water for injection has no pyrogens

degrees of burns

define pharmacoecnomics

bioactivitaion is the activation of prodrug

Page 146: Que & Ans_take Away

amoxicillin is the only chewable antibiotic

transdermal patch should be apply to non hairy area

ionized drug are not reabsorbed

vitamin C is water soluble vitamin also work as an antioxidant

what is ampheprotic

(10)

RESPIRATORY COMPLICATIONS

1- Patient has a relevant increase in his asthma symptoms due to environmental allergies. He is currently having salbutamol and fluticasone inhalers. What would you advise as an adjunct medication to treat his symptoms?

a) Oral corticosteroids

b) Higher dose of corticosteroid inhalers

c) Salmeterol

d) Sodium cromoglycate

e) Anticholinergic agents

2- Treatment of smoke withdrawal symptoms may include which of the following?

I- Nicorette

II- Nicoderm

III- Zyban

a) I only

b) III only

c) I and II only

d) II and III only

e) All are correct

3- Which of the following vitamins is depleted in the blood by smoke?

a) Vitamin C

Page 147: Que & Ans_take Away

b) Vitamin B

c) Vitamin E

d) Vitamin D

e) Vitamin A

4- Condition characterized by a reversible form of airflow obstruction is known as:

a) Aneurism

b) Emphesema

c) Embolism

d) Cirrhose

e) Jaundice

5- Advantages of nasal route over oral route absorption of drugs may include:

I- Nasal route has a higher surface area

II- Nasal route has more blood vessels

III- Nasal route has a higher onset of absorption

a) I only

b) III only

c) I and II only

d) II and III only

e) All are correct

6- Which is not a drug used for smoke cessation?

a) Nicotine polacrilex

b) Transdermal nicotine patches

c) Nicotine sublingual tablets

d) Bupropion

e) Buspirone

Page 148: Que & Ans_take Away

7- Example of leukotriene antagonist drug include:

a) Salbutamol

b) Ipratropium

c) Zafirlukast

d) Montelukast

e) C and D are right

8- Which reaction can ozone cause in the lungs?

a) Oxidation

b) Hydration

c) Reduction

d) Complexation

e) Hydrolysis

9- Correct statements regarding glossitis include which of the following?

I- Inflammation of the tongue

II- Characterized by presence of pinkish-red central lesions on the dorsal surface of the tongue

III- It can be a chronic condition of the tongue seen in pernicious anemia.

a) I only

b) III only

c) I and II only

d) II and III only

e) All are correct

10- Intal-Sodium cromoglycate is characterized by which of the following statements?

I- Non-steroidal drug with anti-inflammatory proprieties

II- They can be used in prophylaxis of asthma and allergic rhinitis

III Used in asthma only

a) I only

b) III only

c) I and II only

d) II and III only

Page 149: Que & Ans_take Away

e) All are correct

11- Drug of choice in the treatment of chronic obstructive pulmonary disease may include:

a) Salbutamol

b) Ipratropium

c) Salmereol

d) Sodium cromoglycate

e) Fluticasone

12- Chronic progressive lower airway destruction causing reduced pulmonary inspiration and expiratory capacity is known as:

a) Asthma

b) Bronchitis

c) GERD

d) COPD

e) CHF

13- Asthma is a condition of respiratory tract that may be aggravated by:

a) Allergens

b) Cold weather

c) Execise

d) Emotion

e) All are correct

14- Which of the following is considered as the best treatment of asthma?

a) β2-agonists

b) β-Blockers

c) α-agonists

d) Anticholinergic drugs

e) Cholinergic drugs

15- Which of the following agents or combinations may be appropriated to treat severe intermittent asthma?

a) High doses of inhaled corticosteroid

b) Long acting β2-agonists

Page 150: Que & Ans_take Away

c) Short acting β2-agonists

d) Oral corticosteroids

e) Combination of all the above medication

16- Examples of drug(s) that may precipitate asthma include all of the following, EXCEPT:

a) Aspirin

b) β-Blockers

c) Ipratropium

d) NSAIDs

e) Cholinergic drugs

17- All of the following are pathophysiologic characteristics of asthma, EXCEPT:

a) Airway obstruction and inflammation

b) Destructive changes in the alveolar walls

c) Thickened smooth muscle of airway

d) Mucosal edema

e) Narrowed lumen of airway

18- The MOST well known characteristic symptom of asthma include:

a) Wheezing

b) Mucosal edema

c) Cough

d) Chest tightness

e) Tachycardia

19- Drug considered as the drug of choice in a specific asthma emergency treatment include:

a) Salmeterol

b) Adrenaline

c) Albuterol

d) Cortisone

e) Zafirlukst

20- Salbutamol is short acting β2-agonists that may be used in the treatment of which of the following asthma type?

Page 151: Que & Ans_take Away

a) Mild asthma

b) Emergency asthma

c) Severe intermittent asthma

d) Moderate asthma

e) All kind of asthma

21- All are correct concerning the action of corticosteroids in asthma status, EXCEPT:

a) Suppress the inflammatory response

b) Decrease production of inflammatory mediators

c) Decrease airway responsiveness to inflammation

d) Relieve brochocontriction

e) Increase β-agonist receptors response

22- All are examples of corticosteroids inhalers used in asthma, EXCEPT:

a) Beclometasone

b) Sodium cromoglycate

c) Flunisolide

d) Fluticasone

e) Budesonide

23- All are examples of drugs used in the treatment of bronchoconstriction, EXCEPT:

a) β2-agonists

b) Anticholinergic

c) Methyl xanthenes

d) Xanthine oxidase inhibitors

e) Leukotriene modifiers

24- Best drug used in the prevention of exercise-induced asthma may include:

a) Long acting β2-agonists

b) Short acting β2-agonists

c) Anticholinergics

d) Leukotriene modifiers

e) Corticoids

Page 152: Que & Ans_take Away

25- All are β2-agonists used as brochodilators in the treatment of bronchospasm, EXCEPT:

a) Albuterol

b) Salmeterol

c) Terbutaline

d) Pindolol

e) Levalbuterol

26- Example of methyl xanthine used in the treatment of respiratory complications include:

a) Theophyllin

b) Terbutaline

c) Ipratropium

d) Zafirlukast

e) Salmeterol

27- Agents considered as precursor of leukotriene include:

a) Tyrosine

b) Cyanide

c) Arachidonic acid

d) Prostaglandins

e) Histamine

28- Indication of leukotrienes modifiers in the treatment of asthma includes which of the following?

I- Acute treatment

II- Prophylaxis

III- Chronic treatment

a) I only

b) III only

c) I and II only

d) II and III only

e) All are correct

Page 153: Que & Ans_take Away

29- Drug known as the new relative cromolyn like drug include:

a) Cromolyn sodium

b) Cromoglycanate sodium

c) Cromolyin disodium

d) Disodium cromoglycanate

e) Nedocromil sodium

1- D

Page 154: Que & Ans_take Away

Comments: Sodium cromoglycanate would be the best choice because it is a safe anti-inflammatory inhaler for children and very useful in treating environmental allergy and asthma prophylaxis.

2- C

Comments: Nicoret and nicoderm are nicotine replacement therapy for smoking cessation, therefore their use avoid the smoke withdrawal effect.

3- A

Comments: Smokers have more tendencies to vitamin C depletetion rather than other vitamins

4- B

Comments: Aneurysm is an abnormal dilatation of a blood vessel while embolism is an obstruction of airway by a mass transported in the circulation. Cirrhoses and jaundice are liver related complications.

5- D

Comments: Although nasal route has a smaller surface area than intestine drugs administrated by nasal route will have faster onset of action due to large quantity of blood vessels in the nasal route.

6- C

Comments: Buspirone is an atypical anxiolitic agent used to treat anxiety disorders.

7- E

Comments: Zafirlukast and montelukast are classics leukotriene antagonists used in the oral treatment of asthma

8- A

Comments: Ozone is a gas that may cause irritation and toxicity of pulmonary system. The main reaction of ozone in the body is oxidation.

9- E

Comments: Glossitis is an inflammatory disease of tongue of unknown etiology characterized by multiple annular areas of desquamation of the filiform papillae on the dorsal surface of the tongue. Usually presenting pinkish-red central lesions, normally seen in pernicious anemia.

10- C

Page 155: Que & Ans_take Away

Comments: Sodium cromoglycanate is an anti-inflammatory, nonsteroid inhaler, mast cell stabilizer used in asthma prophylaxis and as antiallergic agent.

11- B

Comments: Ipratropium is a cholinergic antagonist used as the drug of choice in the treatment of COPD-Chronic Obstructive Pulmonary Disease by blocking the brochoconstriction caused by activation of the parasympathetic nervous system.

12- D

Comments: COPD-Chronic Obstructive Pulmonary Disease

13- E

Comments: Asthma is a chronic inflammatory disorder of the airway that may be induced by allergens, occupational exposure, exercise, emotion, weather, drugs and so on.

14- A

Comments: Inhalated short-acting β-agonists are the most effective drugs available for all the kinds of asthma treatment.

15- E

Comments: Severe intermittent asthma treatment is best done by short and long-acting β-agonists plus inhaled and oral corticosteroids.

16- C

Comments: Ipratropium is an anticholinergic drug that be used in adjunct treatment of asthma.

17- B

Comments: Destructive changes in the alveolar walls are characteristics of a respiratory complication called “Emphysema”

18- A

Comments: Mainly symptoms of asthma are: Wheezing, shortness of breathing, chest tightness, cough, tachycardia, tachypnea and pulses paradoxus.

19- C

Comments: Albuterol is the fastest acting β-agonist used in the treatment of bronchocostriction emergencies-asthma.

Page 156: Que & Ans_take Away

20- E

Comments: Salbutamol is a fast acting β-agonist and considered the drug of choice in all kind of asthma.

21- D

Comments: The use of corticosteroids in asthma suppresses the inflammatory response and decreases the airway hyperesponsiveness by decreasing the production of inflammatory mediators and mucus production.

22- B

Comments: Sodium cromoglycanate is an anti-inflammatory, antiallergenic and mast cell stabilizer drug.

23- B

Comments: Xanthine oxidase inhibitor is mainly used in the treatment of gout.

24- B

Comments: Salbutamol, a fast acting β-agonist, is the most effective drug in the prevention of exercise-induced asthma.

25- D

Comments: Pindolol is a β-blocker used mainly in the treatment of hypertension.

26- A

Comments: Ipratropium is an anticholinergic drug used in COPD as the drug of choice.

27- C

Comments: Cysteinyl leukotrienes are products of arachidonic acid metabolism. Leukotriene modifiers drugs inhibit leukotriene synthesis by inhibiting 5-lipoxygenase that catalyzes the conversion of arachidonic acid to leukotriene.

28- D

Comments: Leukotriene modifiers are approved for oral prophylaxis and chronic treatment of asthma.

29- E

Page 157: Que & Ans_take Away

Comments: Nedocromil sodium is the new relative cromolyn like drug acting in a very similar way as sodium cromoglycanate does.

(11)

TOXICOLOGY

1.Antidote for mercury may include

a)Dimercaprol

b)EDTA

c)Detoroximine

d)Naloxone

e)succimer

2.Which of the ff is benzodiazepine antagonist

a)naloxone

b)physostigmine

c)flumazenil

d)naltrexone

e)penicillamine

3.What would be the best treatment for cyanide poisoning

a)antivena

b)sodium thiosulfate

c)acetylcycteine

d)oxygen therapy

e)sadium bicarbonate

4.carbon monoxide can be highly toxic because it can easily binds with

Page 158: Que & Ans_take Away

I.hemoglobin

II.myoglobin

III.cytochrome oxidase

Ans:

a)I only

b)III only

c)I & II

d)II & III

e)All of the above

5.Acetaminophen toxicity most well known complication is

a)cardiovascular failure

b)pulmonary edema

c)CNS lethargy

d)Liver necrosis

e)Sedation

6.Salicylate poisoning treatment include which of the following

I.Alkalinizaiton of urine with sadium bicarbonate

II.Decontamination with syrup of IPECAC until 30 mins overdose

III.Charcoal every 6 hours

Ans: a) I only

b)III only

c)I & II

d)II & III

e)All of the above

7.Hyperphopotemia can be best treated with

a)Magnesium Hydroxide

b)Calcium Carbonate

Page 159: Que & Ans_take Away

c)Aluminum Hydroxide

d)Sodium bicarbonate

e)Sodium phosphate

8.Correct statement regarding Ipecac may include

I.It has an emetic effect when large dose are administered

II.It has an expectorant effect when small doses are administered

III.Uses in decontamination procedure during toxic treatment.

Ans . All of the above

9.The organophosphate commonly found in insecticeds are thought to act by which mechanism

a)Combining with acetylcholine

b)Potentiating the action of acethycholinesterase

c)Forming a very stable complex with

d)Reaching at the cholinergic receptor

e)Preventing release of ecetylcholine from nerve ending

10.All of the ff. Can be recognised as Atropine poisoning except:

a)Dry skin

b)Mydriasis

c)Flushed appearance

d)Diarrhea

e)Delirium

11.Antidote for Atropine poisoning include

a)Physostigmine

b)Aminophylline

Page 160: Que & Ans_take Away

c)Pralidoxine

d)Flunazeril

e)Dimercaprol

12.Poison ivy can be treated by which of the ff. Agents

a)Topical antipruritic

b)Antihistamines

c)Systemic corticosteroids

13.A patient is stabilized with heparin as . After 1 hr of administration he exhibited bleeding from his gums, while he was brushing his teeth. The pharmacist can advice him to take

a)Ascorbic Acid

b)Vitamin K

c)Protamine sulfate

d)Quinolone

14.N-actyly cysteine may be the antidote of choice for which of the following agents

a)Aspirin

b)Warfarin

c)Digoxin

d)Lead

e)Acetaminophen

Ans: All of the above

15.Dimercaprol + mercury combination include

I.Dimercaprol best antidote forelemental mercury inhalation poisoning

II.They form stable complex excreted renally

Page 161: Que & Ans_take Away

III.Penicillamine is alternative when mercury GI absorption

Ans. All of the above

16.Naloxone is best antidote for which of the ff overdose agents

a)opioid

b)heavy metal

c)benzodiazepine

17.Management if toxicity of digoxin toxicity may include

I.May cause heart failure. Cardiac disrythmia, N, V, anorexia & confusion

II.Treated by decontamination of supportive therapy

III.Treated with digoxin specific overdose known as FAB antibodies

Ans: All of the above

18.Death due to cyanide poisoning

a)Cyanide - RBC complex formation

b)Cyanide – HB complex formation

c)Cyanide inhibiting cytochrome oxidase

d)Cyanide increasing Hb levels

e)Coronary vessel oclusion

19.The toxicity of methyl alcohol due to formation

a)ketones

b)formaldehyde

c)free alcohol radicals

d)ethylene

e)aldehyde

Page 162: Que & Ans_take Away

20.Acetaminophen toxicity is due to

a)oxidation stress

b)active metabolite

c)free radical chain

d)reactive metabolite

e)reduction of metabolite

21. Salicylate toxicity excessive respiration is due to

I.Excessive production of CO2

II.Pulmonary inhalation

III.Central stimulation in the brain

22.Grayish mouth & loose of teeth are toxic symptoms of

a)Fe salts poisoning

b)Cu poisoning

c)Pb poisoning

23.Vomiting is contraindicated in poisoning is due to:

I.Bleaching

II.Gasoline

III.Light petroleum

24.Universal antidote is a mixture of

Page 163: Que & Ans_take Away

a)Activated charcoal

b)Mg Oxide

c)Tannic Acid

25.Amphetamine overdose can be best treated with

a)Salicylates

b)Benzodiazepines

c)Barbiturates

d)Naloxone

e)Vit B & C

26.True statement regarding treatment of hyperkalemia

I.If change in ECG is detected give Ca to counteract the excess of K on heart

II.Bicarbonate & insulin administration can shift K from extra to intracellular

III.Enema of kayexolate (exchange resin) as dialysis helps to remove excess of K from the body

27.Cardiovascular drug that overdose can cause cyanide intoxication include

a)Nitrates

b)Nitroglycerin

c)Nitroprusside

d)Isosorbide

e)Isosorbide dinitrate

28.Lead poisoning can be treated by

a)BAL – Dimercaprol

b)EDTA

c)Penicillamine

Page 164: Que & Ans_take Away

30.Atropine used as antidote for

a)organophosphate

b)heavy metals

c)salicylates

d)iron

31.Treatment of tricyclic antidepressant dose can be done by all of the ff except

a)Control seizure & cardiotoxicity

b)Addition of phenytoin or benzodiazepine to control seizure

c)Activated charcoal can be used

d)MAO antidepressant can help to overcome patient seizure

e)Control acidosis with administration of sodium bicarbonate

32.Specific antidote for snake bites

a)Detroxamine

b)Aucord

c)Antivenom

d)Naloxone

33.Best treatment for carbon monoxide poisoning

a)alkalinization of urine

b)Oxygen therapy

c)Supportive therapy

d)Acidification of urine

e)Gastric lavage

34.Isoniazide antidote

a)Vitamin A

Page 165: Que & Ans_take Away

b)Vitamin B6

c)Vitamin B12

d)Folic acid

e)Vitamin E

35.Warfarin antidote

a)Vit K

b)Heparin

c)Protamine

d)Vit E

36.Major first step in intoxication & poisoning in the supportive care treatment that involve

a)Detoxification

b)Decontamination

c)Hydration

d)Toxicology lab test

e)Evaluation & support of vital treatment (airway, breath, circulation A,B,C)

37.management in treating patient with depressed mental status include

I.Treat hypoglycemia with 50mL of dextrose 50%

II.Administration of thiamine 100mg IV push

III.Administration of naloxone

38.Decontamination includes

a)Gastric lavage

b)Emesis

Page 166: Que & Ans_take Away

c)Administration of Activated charcoal

d)Administration of adsorbant

e)All of the above

39.Intravenous calcium administration is used in which situation

a)verapamil overdose

b)cocaine overdose

c)verapamil overdose & kyperkalemia

d)hyperkalemia

e)nifedipine overdose

ANSWERS TOXICOLOGY

1A

2C

3B

4E

5D

6E

7C

8E

9C

10D

11A

12E

13C

14E

15E

Page 167: Que & Ans_take Away

16A

17E

18B

19B

20D

21A

22B

23E

24E

25E

26E

27C

28E

29B

30A

31D

32C

33B

34B

35A

36E

37E

38E

39C